Download as pdf or txt
Download as pdf or txt
You are on page 1of 126

All India Mock Test 02 Report Print this page

Question 1. + 2.0 - 0.66

The opening and last sentences of the Preamble, “We, the people……………adopt, enact
and give to ourselves this Constitution”, signifies:

A. the sovereign nature of the State

  B. the political power is ultimately vested with the people

C. the idea of welfare State

D. the government is limited by the Constitution

Question Analytics
4814 USERS 3327 USERS 1487 USERS 79.1 SECS

ATTEMPTED  SOLVED CORRECTLY  SOLVED INCORRECTLY  YOUR TIME

63.58 SECS 0.0 SECS

 AVG. SOLVING TIME  FASTEST SOLVING TIME

Explanation :

The Preamble of the Constitution of India serves as a brief introductory


statement that sets out the guiding purpose, principles and philosophy of the
Constitution.
The idea of the following things are given by the Preamble which includes:
Source of the Constitution
Nature of Indian State
A statement of its objectives
Date of its adoption
The opening and last sentences of the preamble: “We, the people... adopt, enact
and give to ourselves this Constitution” signifies the power is ultimately vested
in the hands of the people.

Therefore, option (b) is the correct answer.

Relevance: On the 75th Republic Day, the Government of India shared on social media
a copy of the original Preamble to the Constitution of India.

Question 2. + 2.0 - 0.66

Consider the following:


1. Procedure established by law
2. Emergency provisions
3. Appointment of Governors

4. Parliamentary form of government

How many of the above features of the Constitution of India have been taken from the
Government of India Act, 1935?

 A. Only one

B. Only two

 C. Only three

D. All four

Question Analytics
4427 USERS 930 USERS 3497 USERS 37.4 SECS

ATTEMPTED  SOLVED CORRECTLY  SOLVED INCORRECTLY  YOUR TIME

57.63 SECS 0.0 SECS

 AVG. SOLVING TIME  FASTEST SOLVING TIME

Explanation :

The Constitution of India draws upon a variety of sources, including historical


documents, legal precedents, constitutions of various other countries, as well as
the Government of India Act, 1935.
The concept of ‘procedure established by law’ enumerated in Article 21
(Protection of Life and Personal Liberty) of the Constitution of India is influenced
by the Japanese Constitution. So, point 1 is not correct.
The provisions related to the features such as the federal scheme, office of the
Governor, judiciary, public service commissions, emergency provisions and
administrative details have been derived from the Government of India Act,
1935.
The emergency provisions have been borrowed from Constitutions of
Weimar (Germany) and Government of India Act, 1935. So, point 2 is
correct.
The Governor of a State in India is appointed by the President of India on the
advice of the Union Cabinet. This feature in the Constitution was borrowed from
the Constitution of Canada. So, point 3 is not correct.
The provisions related to the features namely Parliamentary Government, rule
of law, legislative procedure, single citizenship, cabinet system, prerogative writs,
Parliamentary privileges and bicameralism have been borrowed from the British
Constitution. So, point 4 is not correct.

So, only one of the above features of the Constitution of India have been taken
from the Government of India Act, 1935.

Therefore, option (a) is the correct answer.

Knowledge Box

Government of India Act, 1935 and Emergency Provisions:

The Indian Constitution followed the Government of India Act, 1935 which
embodied emergency provisions in Section 45 for the Centre and in Section 93 for
the Provinces.
It empowered the Chief Executive to declare an emergency. The Act of 1935
was meant to provide provincial autonomy in India. But it also had emergency
provisions which put restrictions on the autonomy of provinces. It involved
questions of relations between centre and provincial units.
The Act empowered the Centre to control and intervene in provincial matters if
emergency arises due to war or internal disturbance and make provision for
carrying out administration if the machinery of ministerial government failed to
function.
In times of emergency due to war or internal disturbances, the Governor
General was armed with power to make laws on all matters, even those in the
provincial list. The Governor General could declare an emergency in provinces.
The Governor General of India had in fact specific responsibility concerning
peace and tranquillity of India or any part thereof. Even he could act
independently in the provincial government through the Governor who
was also empowered to function in the respective provinces and was the
main source of information of the Governor-General.

Question 3. + 2.0 - 0.66

Which one of the following statements is correct?

A. India's dominion status ended with the Indian Independence Act of 1947.

India's dominion status continued until the adoption of the Constitution of


  B.
India in 1950.

C. India remained a dominion until the first general elections in 1951.

D. India's dominion status ended with the withdrawal of British troops in 1949.
Question Analytics
4547 USERS 2905 USERS 1642 USERS 31.6 SECS

ATTEMPTED  SOLVED CORRECTLY  SOLVED INCORRECTLY  YOUR TIME

42.91 SECS 0.0 SECS

 AVG. SOLVING TIME  FASTEST SOLVING TIME

Explanation :

Dominions were autonomous communities within the British Empire which were
“equal in status” but had an “allegiance to the Crown”.
It meant that King George VI continued to reign as the Emperor of India and
Lord Mountbatten was the first Governor-General of the country after
independence.
Jawahar Lal Nehru was sworn in as the Prime Minister and unelected Indian
nationalist leaders were administered oaths in the name of the British King-
Emperor.
Though Indian Independence Day is celebrated on August 15, 1947, in reality, that
was the moment when India and Pakistan acquired dominion status. While
India remained a dominion till 1950, Pakistan retained the status till 1956.
It meant that a British field marshall led the Indian army and judges appointed by
the British continued to be part of the High Courts and the Federal Court.
It was only on January 26, 1950, that India broke free from the monarchy and
transformed into a sovereign democratic republic after its Constitution came
into effect.

Therefore, option (b) is the correct answer.

Question 4. + 2.0 - 0.66

Which of the following provisions of the Constitution of India directly embody social
justice as its principle?

1. Article 21A
2. Article 23
3. Article 38

4. Article 48A
5. Article 50

Select the correct answer using the code given below:


A. 1 and 2 only

 B. 1, 2 and 3 only

C. 3, 4 and 5 only

 D. 1, 2, 3, 4 and 5

Question Analytics
3946 USERS 2434 USERS 1512 USERS 58.7 SECS

ATTEMPTED  SOLVED CORRECTLY  SOLVED INCORRECTLY  YOUR TIME

70.2 SECS 0.0 SECS

 AVG. SOLVING TIME  FASTEST SOLVING TIME

Explanation :

Social justice holds that all people should have equal access to wealth, health,
well-being, privileges and opportunity.
The Constitution of India has ingrained the spirit of social justice into its
provisions. The Preamble mentions 3 kinds of justice- social, political and
economic. All of these are interconnected. Social justice and equality are
complementary to each other. The 'welfare state' concept & socialistic features of
the Constitution and Indian economy are also a step towards social justice.
Many Fundamental Rights and Directive Principles of State Policy embody the
principle of social justice. Some of them are:
Right to Equality - Article 14, 16, 17 and 18
Right to Education - Article 21A. So, point 1 is correct.
Cultural and Educational Rights - Article 29 and 30
Right against Exploitation - Article 23 and 24. So, point 2 is correct.
Promotion of social welfare - Article 38. So, point 3 is correct.
Distribution of resources as per common good - Article 39
Just and humane working conditions - Article 42
Promotion of educational and economic interests of the Scheduled castes,
Scheduled tribes and other weaker sections - Article 46
Article 48A states that ‘’The State shall endeavour to protect and improve the
environment and to safeguard the forests and wildlife of the country.’’ So, point 4
is not correct.
Article 50 of the Constitution of India states that ‘’the State shall take steps to
separate the judiciary from the executive in the public services of the State.’’
Neither of these provisions directly promote the idea of social justice. So, point 5
is not correct.

Therefore, option (b) is the correct answer.

Relevance: Recently, the World Day of Social Justice was celebrated.

Question 5. + 2.0 - 0.66

Which one of the following is not a feature of ‘asymmetric federalism’ in India?

A. Existence of All India Services

B. Flexibility in Constitutional Amendment

C. Integrated nature of Judiciary

  D. Seventh Schedule of the Constitution of India

Question Analytics
4393 USERS 1673 USERS 2720 USERS 29.5 SECS

ATTEMPTED  SOLVED CORRECTLY  SOLVED INCORRECTLY  YOUR TIME

70.11 SECS 0.0 SECS

 AVG. SOLVING TIME  FASTEST SOLVING TIME

Explanation :

Asymmetric federalism refers to federalism based on unequal powers and


relationships in political, administrative and fiscal spheres between the units
constituting a federation. Asymmetry in the arrangements in a federation can be
viewed in both vertical (between Centre and States) and horizontal (among the
States) senses.
Even when the Constitution guarantees near equal powers to the States, in the
overall federal system, the Centre dominates in political, administrative as well
as fiscal spheres. Examples of the asymmetric nature of federalism in India are:
All-India Services: In India, the Centre and the States have their separate
public services. However, there are All-India Services which are common
to both the Centre and the States. The members of these services are
recruited and trained by the Centre which also possesses ultimate control
over them. Thus, these services violate the principle of federalism under
the Constitution.
Flexibility in the Constitutional Amendment: The process of
Constitutional amendment is less rigid in India in comparison to what is
found in other federations. The bulk of the Constitution can be amended by
the unilateral action of the Parliament, either by a simple majority or by a
special majority. Further, the power to initiate an amendment to the
Constitution lies only with the Centre. In the United States of America (USA),
the States can also propose an amendment to the Constitution.
Integrated Judiciary: The Constitution has established an integrated judicial
system with the Supreme Court at the top and the State High Courts below
it. This single system of courts enforces both the Central laws as well as the
State laws. In the USA, on the other hand, there is a double system of courts
whereby the federal laws are enforced by the federal judiciary and the state
laws by the state judiciary.
Division of Powers: The Constitution of India has divided the powers between the
Centre and the States in terms of the Union List, State List and Concurrent List
in the Seventh Schedule. The Union List consists of 98 subjects (originally 97), the
State List 59 subjects (originally 66) and the Concurrent List 52 subjects (originally
47). Both the Centre and the states can make laws on the subjects of the concurrent
list, but in case of a conflict, the Central law prevails. The residuary subjects (ie,
which are not mentioned in any of the three lists) are given to the Centre. It is a
federal feature.

Therefore, option (d) is the answer.

Relevance: Recently, the Kerala Finance Minister criticised the Union Government for
imposing financial disparities on Kerala, emphasising the importance of cooperative
federalism.

Question 6. + 2.0 - 0.66

Which of the following limits the authority and jurisdiction of the Parliament?

1. Preamble
2. Seventh Schedule
3. Judicial Review
4. Fundamental Rights

Select the correct answer using the code given below:

A. 1 and 3 only

B. 2 and 4 only

C. 1, 3 and 4 only
  D. 2, 3 and 4 only

Question Analytics
4688 USERS 3368 USERS 1320 USERS 17.6 SECS

ATTEMPTED  SOLVED CORRECTLY  SOLVED INCORRECTLY  YOUR TIME

52.8 SECS 0.0 SECS

 AVG. SOLVING TIME  FASTEST SOLVING TIME

Explanation :

The Parliament in India is not a sovereign body, it functions within the bounds of a
written Constitution. In India, unlike the United Kingdom, we have a written
constitution which puts necessary limitations on all organs of the state. Parliament
can amend some parts of the Constitution but only to the degree bestowed upon it by
the Constitution.

Preamble: It is neither a source of power to legislature nor a prohibition upon


the powers of legislature. It is non-justiciable, that is, its provisions are not
enforceable in courts of law. So, point 1 is not correct.
Seventh Schedule: Under Article 246 of the Constitution, this Schedule deals with
the division of powers between the Union and the States. It divides the legislative
subjects into Union, Concurrent and State lists. The Parliament is explicitly
prohibited from legislating on matters in the State List. So, point 2 is correct.
Judicial Review: The Indian Parliament, being the architect of the Constitution,
doesn’t enjoy unchecked authority. For a law to be considered valid, it must
undergo judicial review. If a law is found to be inconsistent with the constitutional
provisions, the court has the authority to declare it void. This means that the
judiciary assesses the constitutionality of legislative enactments, limiting
parliamentary sovereignty in India. So, point 3 is correct.
Fundamental Rights: It guarantees Fundamental Rights, which are justiciable,
meaning they can be enforced through legal proceedings. Parliament is expressly
restricted from enacting laws that violate these Fundamental Rights. So,
point 4 is correct.

Therefore, option (d) is the correct answer.


Relevance: The Vice-President recently questioned the Basic Structure Doctrine on the
grounds that it limits Parliamentary authority.

Question 7. + 2.0 - 0.66


Consider the following statements with reference to the making of the Constitution of
India:
1. It is handwritten in both English and in Hindi.

2. Rani Lakshmibai of Jhansi is the only female figure illustrated prominently in the
Constitution.

3. The Bull Seal found in the Indus Valley is the first pictorial representation in the
Constitution.
How many of the above statements are correct?

A. Only one

B. Only two

 C. All three

 D. None

Question Analytics
3711 USERS 529 USERS 3182 USERS 32.1 SECS

ATTEMPTED  SOLVED CORRECTLY  SOLVED INCORRECTLY  YOUR TIME

47.3 SECS 0.0 SECS

 AVG. SOLVING TIME  FASTEST SOLVING TIME

Explanation :

The Constitution of India is not just the lengthiest written Constitution in the world
but also the most richly illustrated, with all 22 parts carrying hand-painted
images and its pages adorned with elaborate borders.
Two copies of the Constitution, one in English and one in Hindi, are
handwritten and bear the paintings. While the Constitution was
handwritten by calligrapher Prem Behari Narain Raizada, the paintings
were conceived and implemented in Santiniketan by artist-pedagogue
Nandalal Bose and his team. So, statement 1 is correct.
The Preamble page has intricate patterns sketched by Beohar Rammanohar
Sinha and bears his signature, while Dinanath Bhargava sketched the
National Emblem, the Lion Capital of Ashoka.
The only female figure illustrated prominently in the Constitution, Rani
Lakshmibai of Jhansi, is sketched in her armour as she shares the page with Tipu
Sultan, the king of Mysore, in Part XVI of the Constitution. Chhatrapati Shivaji
and Guru Gobind Singh are featured in Part XV. So, statement 2 is correct.
The Bull Seal, excavated from the Indus Valley region, is the first pictorial
representation in the Constitution, appearing in ‘Part I: The Union and its
Territory’. So, statement 3 is correct.

So, all three of the above statements are correct.


Therefore, option (c) is the correct answer.

Relevance: India celebrated Samvidhan Divas, also known as Constitution Day, on


November 26 to commemorate the adoption of the Constitution in 1949.

Question 8. + 2.0 - 0.66

Consider the following statements:

Statement-I: The provisions of the Tenth Schedule, with respect to voting against the
instruction of the party, do not apply to Rajya Sabha elections.
Statement-II: The Constitution of India provides that the voting in elections to the Rajya
Sabha shall be through an open ballot.
Which one of the following is correct in respect of the above statements?

Both Statement–I and Statement–II are correct and Statement–II is the


A.
correct explanation for Statement–I

Both Statement–I and Statement–II are correct and Statement–II is not the
 B.
correct explanation for Statement–I

 C. Statement–I is correct but Statement–II is incorrect

D. Statement–I is incorrect but Statement–II is correct

Question Analytics
4198 USERS 1642 USERS 2556 USERS 62.3 SECS

ATTEMPTED  SOLVED CORRECTLY  SOLVED INCORRECTLY  YOUR TIME

66.78 SECS 0.0 SECS

 AVG. SOLVING TIME  FASTEST SOLVING TIME

Explanation :

The 52nd Constitutional Amendment Act of 1985 introduced the ‘anti-defection’


law and added the Tenth Schedule to the Constitution of India. This Schedule
provides that a member of a House of Parliament or State Legislature who
voluntarily gives up the membership of their political party or votes against the
instructions of their party in a House are liable for disqualification from such
House. This instruction with respect to voting is issued by the ‘whip’ of a party.
However, the elections to Rajya Sabha are not treated as a proceeding within
the Legislative Assembly. The Election Commission of India, drawing
reference to Supreme Court judgments, had specified that the provisions of
the Tenth Schedule, with respect to voting against the instruction of the
party, will not be applicable for a Rajya Sabha election. Furthermore,
political parties cannot issue any ‘whip’ to its members for such elections. So,
statement 1 is correct.
In order to control the Members of Legislative Assemblies (MLAs) from cross-
voting, an amendment to the Representation of the People Act, 1951 (and not
the Constitution of India) was carried out in 2003. It provided that the voting in
elections to Rajya Sabha shall be through an open ballot. The MLAs of political
parties are required to show their ballot paper to the authorised agent of their
Party. Not showing the ballot paper to the authorised agent or showing it to
anyone else will disqualify the vote. Independent MLAs are barred from showing
their ballots to anyone. So, statement 2 is not correct.

So, statement–I is correct but Statement–II is incorrect.


Therefore, option (c) is the correct answer.
Relevance: Recently, the Speaker of the Maharashtra Legislative Assembly refused to
disqualify 16 Members of the Legislative Assembly (MLAs).

Question 9. + 2.0 - 0.66

A ‘Parliamentary democracy’ is characterised by:

A. strict separation of the executive and legislative organs

B. appointment of the executive branch by the legislature

C. direct elections of the representative to the executive office

  D. fusion of executive and legislative powers into a single body

Question Analytics
4632 USERS 1173 USERS 3459 USERS 56.4 SECS

ATTEMPTED  SOLVED CORRECTLY  SOLVED INCORRECTLY  YOUR TIME

58.42 SECS 0.0 SECS

 AVG. SOLVING TIME  FASTEST SOLVING TIME

Explanation :
Parliamentary democracy is characterised by the fusion of executive and
legislative powers into a single body. The executive, the Cabinet Ministers, sit as
members of Parliament and play a double role in the fusion of executive and
legislative powers.
Those who recommend legislation as members of the Cabinet also vote on the
same legislation as members of the governing legislature. Typically, the
government is the Cabinet and the rest of the members of the majority party or
coalition of parties in the Parliament.
Parliamentary democracy means collegial executive. Although the Prime
Minister is the chief executive, he or she is not a singular executive. The collegial
executive is a collection of Ministers (Cabinet) who must make decisions as a
group and must be in general agreement before legislation is recommended or
policies are proposed. The Ministers are both individually and collectively
responsible to the Parliament for their actions. Parliamentary democracy also
means a democracy based on party responsibility.

Therefore, option (d) is the correct answer.

Question 10. + 2.0 - 0.66

Consider the following bills:


1. Money Bill
2. Private Member Bill

3. Constitutional Amendment Bill


The President of India can use her/his Pocket Veto with respect to how many of the
above?

  A. Only one

B. Only two

C. All three

D. None

Question Analytics
4653 USERS 1814 USERS 2839 USERS 18.7 SECS

ATTEMPTED  SOLVED CORRECTLY  SOLVED INCORRECTLY  YOUR TIME

41.27 SECS 0.0 SECS

 AVG. SOLVING TIME  FASTEST SOLVING TIME

Explanation :
A Bill passed by the Parliament can become an Act only if it receives the assent of
the President of India. When a Bill is presented to the President for her/his assent,
s/he has three alternatives (under Article 111 of the Constitution of India):
S/He may give assent to the Bill
S/He may withhold assent to the Bill (Absolute Veto)
S/He may return the Bill (if it is not a Money Bill) for reconsideration of the
Parliament (Suspensive Veto). However, if the Bill is passed again by the
Parliament with or without amendments and again presented to the
President, it is obligatory for the President to give assent to the Bill.
With respect to Pocket Veto, the President neither ratifies nor rejects nor returns
the Bill, but simply keeps it pending for an indefinite period. This power of the
President not to take any action (either positive or negative) on the Bill is known
as the Pocket Veto. The President can exercise this veto power as the Constitution
does not prescribe any time-limit within which he has to take the decision with
respect to a Bill presented to her/him for assent.
With respect to a Money Bill, the President may either give or withhold assent. By
convention, s/he gives the assent to the Bill as it is introduced in the Parliament
based on her/his recommendation. A Money Bill cannot be returned to the House
by the President for reconsideration. So, point 1 is not correct.
The President can withhold or keep it pending or return a Private Member Bill
for the reconsideration of the Parliament. So, point 2 is correct.
The President is bound to give her/his assent to the Constitutional
Amendment Bill passed by Parliament by the prescribed special majority and,
where necessary, ratified by the requisite number of State Legislatures. So, point 3
is not correct.

So, the President of India can use her/his Pocket Veto with respect to only one of
the above bills.
Therefore, option (a) is the correct answer.

Question 11. + 2.0 - 0.66

If the Gross National Product (GNP) of India is more than its Gross Domestic Product
(GDP), it necessarily implies an increase in:

  A. net factor income from abroad

B. depreciation of fixed assets


C. gross fixed capital consumption

D. output produced within India by non-residents

Question Analytics
4502 USERS 3932 USERS 570 USERS 44.2 SECS

ATTEMPTED  SOLVED CORRECTLY  SOLVED INCORRECTLY  YOUR TIME

54.09 SECS 0.0 SECS

 AVG. SOLVING TIME  FASTEST SOLVING TIME

Explanation :

Gross National Product (GNP) is the total value of all finished goods and
services produced by a country’s citizens in a given financial year, irrespective of
their location. It also measures the output generated by a country’s businesses
located domestically or abroad. It does not include foreign residents’ income
earned within the country.
The Gross Domestic Product (GDP) is the final value of goods and services
produced in a given period of time within a country.
The basic distinction between GDP and GNP is the difference in estimating the
production output by foreigners in a country and by nationals outside of a
country. It confines its analysis of the economy to the geographical borders of the
country, GNP extends it to also take account of the net overseas economic activities
performed by its residents. In other words, GNP equals to GDP plus Net Factor
Income From Abroad (NFIA).
NFIA is the difference between factor income earned by the residents who are
temporarily residing abroad and factor income earned by non-residents who are
temporarily residing in our country.

Therefore, option (a) is the correct answer.


Relevance: Recently, the Fitch Ratings raised India's economic growth forecast for next
fiscal to 7 per cent.

Question 12. + 2.0 - 0.66

Consider the following statements:


The money multiplier in the economy will increase if there is:
1. an increase in the reserve requirement by the Central Bank

2. an increase in the currency deposit ratio


3. an increase in the deposit rates by banks
How many of the above statements are correct?

 A. Only one

 B. Only two

C. All three

D. None

Question Analytics
4198 USERS 1569 USERS 2629 USERS 57.7 SECS

ATTEMPTED  SOLVED CORRECTLY  SOLVED INCORRECTLY  YOUR TIME

67.59 SECS 0.0 SECS

 AVG. SOLVING TIME  FASTEST SOLVING TIME

Explanation :

Money multiplier shows the mechanism by which reserve money creates money
supply in the economy. It is dependent on two variables, namely currency deposit
ratio and reserve deposit ratio.
Reserve deposit Ratio, also known as Cash Reserve Ratio, is the percentage of
deposits which commercial banks are required to keep as cash according to the
directions of the Central Bank.
If the reserve requirement as stipulated by the Central Bank increases, the
Reserve Money value will increase and the multiplier will fall. Similarly, if banks
keep more money as excess reserves, it will have an adverse effect on the money
multiplier. So, statement 1 is not correct.
The currency deposit ratio shows the amount of currency that people hold as a
proportion of aggregate deposits. An increase in currency deposit ratio leads to a
decrease in money multiplier. So, statement 2 is not correct.
So, the banks can increase deposit rates to induce depositors to deposit
more money thereby leading to a decrease in cash to deposit ratio. This will
in turn lead to a rise in money multiplier. So, statement 3 is correct.
Increase in the banking habits of the people of the country will lead to more
deposits in the banks. This leads to an increase in money multiplier.

So, only one of the above statements is correct.


Therefore, option (a) is the correct answer.
Relevance: Liquidity conditions have remained tight in the banking system for some
time due to lower government spending, higher tax outflows and slower bank deposit
growth.

Question 13. + 2.0 - 0.66

In an economy, the ‘Effective Capex’ means:

A. capital expenditure less the grant-in-aid for creation of capital assets

 B. capital expenditure less the depreciation value of the capital assets

expenditure incurred in the current accounting year to create revenue in a


C.
future

total capital outlay of the government directed towards building of capital


 D.
assets
Question Analytics
3827 USERS 1091 USERS 2736 USERS 23.1 SECS

ATTEMPTED  SOLVED CORRECTLY  SOLVED INCORRECTLY  YOUR TIME

59.06 SECS 0.0 SECS

 AVG. SOLVING TIME  FASTEST SOLVING TIME

Explanation :

Capital expenditure is the money spent by the government on the development


of machinery, equipment, building, health facilities, education, etc. It also
includes the expenditure incurred on acquiring fixed assets like land and
investment by the government that gives profits or dividends in future.
Effective Capital Expenditure is total capital outlay of the Government
directed towards building of capital assets and related expenditure. It is the
sum of Capital Expenditure and Grants-in-Aid for creation of Capital Assets (GIA
Capital).

Therefore, option (d) is the correct answer.


Relevance: The Union Finance Minister in the Interim Budget of 2023-24 has
announced that the Effective Capital Expenditure of the Centre would be budgeted at ₹
13.7 lakh crore i.e. 4.5% of GDP.

Question 14. + 2.0 - 0.66

Consider the following:


1. Nutrient Based Subsidy
2. Minimum Support Prices (MSP) for essential crops
3. Subsidy for purchase of machinery under centrally-sponsored schemes
4. Free electricity to farmers
How many of the above are considered indirect farm subsidies granted by the
Government of India?

A. Only one

 B. Only two

C. Only three

 D. All four

Question Analytics
4334 USERS 1709 USERS 2625 USERS 27.3 SECS

ATTEMPTED  SOLVED CORRECTLY  SOLVED INCORRECTLY  YOUR TIME

67.05 SECS 0.0 SECS

 AVG. SOLVING TIME  FASTEST SOLVING TIME

Explanation :

Subsidies are provided through various schemes to the agricultural sector by the
Central and State Governments in order to promote the adoption of certain inputs/
machinery etc. in crop cultivation.
Direct subsidies are monetary transfers by the government that reach the
ultimate beneficiary through a formal predetermined route.
The government provides direct subsidy through fixation of Minimum
Support Price (MSP) for essential crops and purchase of machinery, drip
and sprinkler irrigation under various centrally-sponsored schemes. So,
points 2 and 3 are not correct.
Indirect subsidies by the Government are the ones that reach the farmers along
with the use of inputs. Therefore, these are highly correlated with the amount of
use of inputs by farmers. Generally, those farmers who use more inputs would
enjoy higher subsidies.
The indirect subsidy is extended through provision of inputs such as
irrigation, fertiliser (Nutrient Based Subsidy) and power at prices much
below their cost of production (free electricity to farmers). So, points 1
and 4 are correct.

So, only two of the above are considered indirect farm subsidies granted by the
Government of India.
Therefore, option (b) is the correct answer.
Relevance: The Union Minister of Finance recently said that the World Trade
Organisation (WTO) should look at the issue of farm subsidies keeping in mind its
impact on the food security needs of emerging economies in the backdrop of the Russia-
Ukraine war.

Question 15. + 2.0 - 0.66

Consider the following statements with reference to the interim budget:


1. The Constitution of India does not make any explicit provision for the interim budget.
2. It was first presented in the year of the first general election in India.

3. It deals only with the expenditure side of the government.


How many of the above statements are correct?

 A. Only one

 B. Only two

C. All three

D. None

Question Analytics
4092 USERS 1675 USERS 2417 USERS 94.3 SECS

ATTEMPTED  SOLVED CORRECTLY  SOLVED INCORRECTLY  YOUR TIME

51.49 SECS 0.0 SECS

 AVG. SOLVING TIME  FASTEST SOLVING TIME

Explanation :

The interim budget is a temporary financial plan announced before a new


government is set to come in after general elections. It can be understood as a
provisional arrangement to meet the expenditure of the government for a short
period till the new government comes in and makes the new full budget.
An interim budget is similar to a full budget but only has projections for a
few months, as opposed to a full financial year.
There is no constitutional provision for an interim budget. However, it has
become a common practice for outgoing governments ahead of elections. A ruling
government can alternatively choose to obtain the necessary funds for expenses
through the vote-on-account provision under Article 116 of the Constitution
instead of presenting an interim budget. So, statement 1 is correct.
India's first interim budget in 1947 was presented by the first Finance Minister
R.K. Shanmukham Chetty. It addressed economic challenges caused by partition,
including food grain shortage and high imports. So, statement 2 is not correct.
An interim budget contains both the revenue and expenditure details for the
period until the new government takes over. On the other hand, vote-on-account
includes only the government's expenditure. So, statement 3 is not correct.

So, only one of the above statements is correct.


Therefore, option (a) is the correct answer.

Relevance: The Interim Budget 2024 was presented by the Union Finance Minister
recently.

Question 16. + 2.0 - 0.66

Which of the following measures can lead to a decline in Current Account Deficit of
India?
1. Reduction in fertiliser and food subsidies
2. Diversification of the export markets

3. Reduction in customs duties on imports


4. Reduction in transfer costs of remittances
Select the correct answer using the code given below:

A. 1 and 2 only

B. 3 and 4 only

  C. 1, 2 and 4 only

D. 1, 2, 3 and 4

Question Analytics
4128 USERS 2389 USERS 1739 USERS 50.5 SECS

ATTEMPTED  SOLVED CORRECTLY  SOLVED INCORRECTLY  YOUR TIME

89.54 SECS 0.0 SECS

 AVG. SOLVING TIME  FASTEST SOLVING TIME

Explanation :

The Current Account Deficit (CAD) is a measurement of a country’s trade where


the value of imports exceeds the value of its exports. It represents a country’s
foreign transactions and, like the capital account, is a component of its Balance of
Payments (BoP). Several measures can potentially lead to a decline in the CAD of
India:
Fiscal Discipline: Fiscal consolidation measures such as reducing fertiliser
and food subsidies, controlling public expenditure and improving
revenue generation can help in narrowing the CAD. So, point 1 is correct.
Export Promotion: Implementing policies and strategies to boost exports
can increase foreign exchange inflows and reduce the trade deficit, which
is a component of the CAD. It can be achieved through trade promotion
initiatives, improvement in competitiveness, diversification of export
markets and support to industries with export potential. So, point 2 is
correct.
Import Substitution: Encouraging domestic production of goods that are
currently being imported can reduce reliance on imports and subsequently
narrow the trade deficit. Promoting domestic industries, providing
incentives, and improving the business environment can support import
substitution efforts. On the other hand, promotion of imports by means of
reducing customs duty and easing qualitative barriers can lead to widening
of the trade deficit. So, point 3 is not correct.
Increase in Remittances: Encouraging overseas workers to send more
remittances back to their home country can boost the current account
balance. Measures such as facilitating remittance channels, reducing
transfer costs and promoting financial inclusion can incentivize higher
remittance inflows. So, point 4 is correct.
Current Account Management: This can involve controlling non-essential
imports, managing import costs, implementing effective exchange rate
policies, and monitoring capital flows to maintain stability.

Therefore, option (c) is the correct answer.


Relevance: Economists were prompted to lower the Current Account Deficit (CAD) at
below 1% of GDP for FY24 amidst narrowing of India’s trade deficit and increase in
services exports.

Question 17. + 2.0 - 0.66

If the tax multiplier is 1 in an economy, then 1 percent tax cut will lead to:

 A. 1 percent boost to GDP

 B. 10 percent boost to GDP

C. 1 percent reduction in GDP


D. no change in GDP

Question Analytics
2859 USERS 920 USERS 1939 USERS 41.6 SECS

ATTEMPTED  SOLVED CORRECTLY  SOLVED INCORRECTLY  YOUR TIME

55.48 SECS 0.0 SECS

 AVG. SOLVING TIME  FASTEST SOLVING TIME

Explanation :

The tax multiplier is defined as the amount by which a change in taxes affects the
Gross Domestic Product (GDP). This tool allows the government to reduce (raise)
taxes by the exact amount required for GDP to rise (fall). This allows the
government to implement a specific tax change rather than an estimate.
Both public investments and tax cuts have multiplier effects on the economy. The
capex multiplier in India is around 2.5 while the tax multiplier is closer to 1.
So, a one-rupee tax cut can provide only a one-rupee boost to GDP, while every
rupee spent on capex will boost GDP by 2.5x.
The Union Budget of 2024 has balanced economic stimulus with fiscal discipline.
The stimulus comes from a 33% increase in public investments covering roads,
ports, airports and railways. The income tax rejig may also produce higher
demand if people divert their tax saving towards spending.

Therefore, option (a) is the correct answer.

Relevance: Recently, the Union Budget 2024 has announced a 33% increase in public
investments.

Question 18. + 2.0 - 0.66

Consider the following statements:


Statement-I:
The bond prices tend to increase with decrease in the interest rates by the banks.
Statement-II:

Most Bonds pay a fixed interest rate that provides higher returns than the bank
deposits.

Which one of the following is correct in respect of the above statements?

Both Statement-I and Statement-II are correct and Statement-II is the correct
 A.
explanation for Statement-I
Both Statement-I and Statement-II are correct and Statement-II is not the
 B.
correct explanation for Statement-I

C. Statement-I is correct but Statement-II is incorrect

D. Statement-I Is incorrect but Statement-II is correct

Question Analytics
4017 USERS 1838 USERS 2179 USERS 72.4 SECS

ATTEMPTED  SOLVED CORRECTLY  SOLVED INCORRECTLY  YOUR TIME

73.53 SECS 0.0 SECS

 AVG. SOLVING TIME  FASTEST SOLVING TIME

Explanation :

Bonds have an inverse relationship to interest rates. When the cost of


borrowing money rises (when interest rates rise), bond prices usually fall, and
vice-versa. So, statement 1 is correct.
Most bonds pay a fixed interest rate that becomes more attractive if interest
rates fall, driving up demand and the price of the bond. So, statement 2 is
correct.
As interest rates decrease, the present value of the fixed future cash flows from a
bond increases. This is because the discount rate used to calculate present value
(which is based on prevailing interest rates) is lower, leading to a higher present
value.
A higher present value translates to a higher market price for the bond.
Conversely, if interest rates rise, investors will no longer prefer the lower fixed
interest rate paid by a bond, resulting in a decline in its price.
Zero-coupon bonds provide a clear example of how this mechanism works in
practice.

So, both Statement-I and Statement-II are correct and Statement-II is the correct
explanation for Statement-I.
Therefore, option (a) is the correct answer.

Relevance: Recently, Bloomberg data showed that the 10-year benchmark bond yield
closed at 7.01%, its lowest level since June 14, 2023.

Question 19. + 2.0 - 0.66

Consider the following statements with reference to the Total Expense Ratio (TER):
1. It is a measure of the operational efficiency of mutual funds.
2. It is used to calculate the return on investments after consideration of all fees.
3. The lower the expense ratio, the lower the returns on investment.

How many of the above statements are correct?

 A. Only one

 B. Only two

C. All three

D. None

Question Analytics
2465 USERS 1858 USERS 607 USERS 55.9 SECS

ATTEMPTED  SOLVED CORRECTLY  SOLVED INCORRECTLY  YOUR TIME

42.27 SECS 0.0 SECS

 AVG. SOLVING TIME  FASTEST SOLVING TIME

Explanation :

Total Expense Ratio (TER):

It is a measure of the total costs associated with managing and operating an


investment fund, such as a mutual fund. It is also known as the net expense
ratio or after-reimbursement expense ratio. The total cost of the fund is divided
by the fund’s total assets to arrive at a percentage amount, which represents the
TER. It throws light on the operational efficiency of mutual funds. So, statement
1 is correct.
The TER covers various expenses, including management fees, administrative fees,
distribution fees, auditing fees, legal fees, and other operating expenses. It helps
investors understand the overall cost they will bear when investing in a
mutual fund. The TER directly affects the overall return from the mutual fund.
Higher TER means higher expenses. For Example: For any scheme, if its return is
22% and its expense ratio is 2.25%, then the net return for the investor will be
19.75%. So, statement 2 is correct.
It is used by investors to compare the costs of the scheme with its peers and
also about the returns available from that scheme. It is a key element in making
an investment choice. A lower TER could mean a higher profitability rate.
Those funds which consistently show a high TER may not provide high returns,
since high expenses tend to erode the returns generated. So, statement 3 is not
correct.
So, only two of the above statements are correct.
Therefore, option (b) is the correct answer.

Relevance: The Securities and Exchange Board of India has recently decided to issue a
fresh consultation on the issue of Total Expense Ratio (TER) for mutual funds.

Question 20. + 2.0 - 0.66

An increase in the interest rate in an economy will most likely lead to:

A. increase in investment by businesses

  B. higher individual savings

C. increase in consumer spending

D. higher inflation rates

Question Analytics
4573 USERS 3305 USERS 1268 USERS 42.0 SECS

ATTEMPTED  SOLVED CORRECTLY  SOLVED INCORRECTLY  YOUR TIME

52.04 SECS 0.0 SECS

 AVG. SOLVING TIME  FASTEST SOLVING TIME

Explanation :

Higher savings usually occur when interest rates are higher, as individuals
seek better returns on their savings in interest-bearing accounts.
On the contrary, lower interest rates make borrowing cheaper which
encourages businesses to invest in expansion and individuals to take loans for
various purposes.
When interest rates are lowered, it tends to encourage individuals to take out
loans for major purchases such as homes or cars, leading to increased spending.
This contributes to increased money supply in the economy, which can put
upward pressure on prices rather than reducing inflation.
Businesses, with reduced costs of financing, are incentivized to invest in new
projects and initiatives. This contributes to overall economic growth.

Therefore, option (b) is the correct answer.

Relevance: The Reserve Bank of India’s Monetary Policy Committee has decided to keep
the policy repo rate unchanged at 6.5% as retail inflation continues to be above its
target of 4%.
Question 21. + 2.0 - 0.66

Consider the following statements:

1. Hibernation allows animals to increase metabolism during periods of high


temperatures.

2. Aestivation is characterised by lower body temperatures to survive long cold winters.

3. Brumation is the state of dormancy observed in cold-blooded animals during cold


seasons.

How many of the above statements are correct?

 A. Only one

B. Only two

C. All three

 D. None

Question Analytics
3555 USERS 1999 USERS 1556 USERS 24.1 SECS

ATTEMPTED  SOLVED CORRECTLY  SOLVED INCORRECTLY  YOUR TIME

50.57 SECS 0.0 SECS

 AVG. SOLVING TIME  FASTEST SOLVING TIME

Explanation :

Hibernation is a prolonged state of torpor that helps animals to slow down their
heart rate and lower their body temperature for extended periods to survive
long cold winters. Many rodent’s body temperatures reach almost freezing at this
time. The majority of these hibernating animals still have a food store, waking up
every few weeks to eat something and return to hibernate again until they can
wake up properly in spring. So, statement 1 is not correct.
Aestivation is a prolonged state of torpor that animals undergo in the heat or
during drought. The mechanism is similar to hibernation, where organisms use
their physiology to slow their metabolism way down and enter a deep sleep,
reducing the need for food and especially water. This happens in places like
deserts but has also been found in marine habitats, where marine animals go
more dormant as water temperature is higher and food availability is low. So,
statement 2 is not correct.
Brumation is the state of dormancy or slowed metabolic activity observed in
certain ectothermic (cold-blooded) animals, particularly reptiles, during colder
months. It is also a form of prolonged hypometabolism, but brumation is not
torpor because it is a type of hypometabolism undergone by ectothermic animals.
These ectotherms simply allow their environment to cool their body down and
slow down their metabolism. Common examples include snakes, lizards, turtles
and certain types of frogs. So, statement 3 is correct.

So, only one of the above statements is correct.

Therefore, option (a) is the correct answer.

Knowledge Box

Torpor or cold-lethargy is a hypometabolic state (very low metabolism)


associated with low body temperature. It is used by endothermic animals to save
energy to survive harsh conditions such as cold temperatures or lack of food.
Daily torpor is distinct from hibernation and aestivation, which are considered
prolonged torpor.
Hummingbirds are a great example of daily torpor in animals, going into a deep
sleep state every single night. Animals undergoing daily torpor have an extremely
high metabolism and need to feed every few minutes, so at night, they reduce
their heart rate and body temperature to rest properly. It can also apply to
animals that in general have poor diets, such as koala bears and sloths. Animals
in deserts like cactus mice (Peromyscus eremicus) undergo daily torpor, to avoid
extreme heat during the day.

Relevance: Experts suggest that hibernation may hold important clues on how to slow
down ageing processes.

Question 22. + 2.0 - 0.66

Consider the following:


1. Methane

2. Nitrous oxide

3. Chlorofluorocarbons
4. Ozone

5. Sulfur hexafluoride
How many of the above greenhouse gases occur naturally in the atmosphere?

 A. Only two

 B. Only three
C. Only four

D. All five

Question Analytics
4467 USERS 2480 USERS 1987 USERS 97.6 SECS

ATTEMPTED  SOLVED CORRECTLY  SOLVED INCORRECTLY  YOUR TIME

46.18 SECS 0.0 SECS

 AVG. SOLVING TIME  FASTEST SOLVING TIME

Explanation :

Greenhouse gas refers to any gas that has the property of absorbing infrared
radiation emitted from the Earth’s surface and re-radiating it back to the Earth’s
surface leading to an increase in temperature, thus contributing to the greenhouse
effect.

Some Greenhouse Gases (GHGs) occur naturally in the atmosphere, while others
result from human activities. Naturally occurring greenhouse gases include:
Water Vapour
Carbon dioxide
Methane. So, point 1 is correct.
Nitrous oxide. So, point 2 is correct
Ozone. So, point 4 is correct.
Many synthetic, powerful greenhouse gases are emitted from a variety of
household, commercial, and industrial applications and processes. Some of them
are:
Hydrofluorocarbons
Perfluorocarbons
Nitrogen trifluoride
Chlorofluorocarbons. So, point 3 is not correct.
Sulfur hexafluoride. So, point 5 is not correct.

So, only three of the above greenhouse gases occur naturally in the atmosphere.
Therefore, option (b) is the correct answer.

Relevance: As per United Nations’s World Meteorological Organisation, greenhouse


gases have hit record high in 2023.

Question 23. + 2.0 - 0.66


In India, no alteration of the boundaries of a National Park by the State Government
can be made except on a recommendation of:

 A. National Board for Wildlife (NBWL)

 B. National Green Tribunal (NGT)

C. National Biodiversity Authority (NBA)

D. Animal Welfare Board of India (AWBI)

Question Analytics
4411 USERS 3085 USERS 1326 USERS 59.3 SECS

ATTEMPTED  SOLVED CORRECTLY  SOLVED INCORRECTLY  YOUR TIME

32.98 SECS 0.0 SECS

 AVG. SOLVING TIME  FASTEST SOLVING TIME

Explanation :

In India, the Wildlife Protection Act, 1972 is a key legislation that governs the
conservation and protection of wildlife and their habitats. Under this Act, the
establishment, management and regulation of National Parks fall under the
purview of the State Government.
To ensure that such decisions related to any alteration to the boundaries of a
National Park are made with due diligence and expertise, the above Act mandates
that any alteration to the boundaries of a National Park must be
recommended by the National Board for Wildlife (NBWL).
The NBWL is a statutory organisation constituted under the Wildlife
Protection Act, 1972, chaired by the Prime Minister of India and comprises
eminent conservationists, scientists and government officials.
By involving the NBWL in the decision-making process, there is an added layer of
scrutiny and evaluation to ensure that changes to National Park boundaries are in
line with conservation objectives to minimise adverse impacts on the environment
and wildlife.

Therefore, option (a) is the correct answer.

Relevance: The National Board for Wildlife has accepted two elevated railway lines
surrounding Deepor Beel lake in Assam.

Question 24. + 2.0 - 0.66

Consider the following statements:


1. Global Tiger Initiative (GTI) was launched by the World Bank and Global
Environment Facility.
2. Global Tiger Forum (GTF) is an intergovernmental body including all tiger range
countries.
3. Conservation Assured Tiger Standards (CA|TS) is a global tool that sets minimum
standards for management of tigers.

4. Only 14 Tiger Reserves in India have received the CA|TS accreditation.


How many of the above statements are correct?

A. Only one

 B. Only two

 C. Only three

D. All four

Question Analytics
2555 USERS 1134 USERS 1421 USERS 136.9 SECS

ATTEMPTED  SOLVED CORRECTLY  SOLVED INCORRECTLY  YOUR TIME

43.67 SECS 0.0 SECS

 AVG. SOLVING TIME  FASTEST SOLVING TIME

Explanation :

The Global Tiger Initiative (GTI) was launched by the World Bank, Global
Environment Facility, Smithsonian Institution, Save the Tiger Fund, and
International Tiger Coalition. The GTI is led by the 13 tiger range countries. It is
a global alliance of governments, international organisations, civil society, the
conservation and scientific community and the private sector that are committed
to working together toward a common agenda to save wild tigers from extinction.
So, statement 1 is correct.
The Global Tiger Forum (GTF) is an international intergovernmental body
exclusively set up for the conservation of tigers in the wild in certain tiger range
countries. Out of the 13 tiger range countries, seven are currently members of
GTF: Bangladesh, Bhutan, Cambodia, India, Myanmar, Nepal and Vietnam besides
non-tiger range country, the United Kingdom. The secretariat is based in New
Delhi, India. So, statement 2 is not correct.
Conservation Assured Tiger Standards (CA|TS) is a globally accepted
conservation tool that sets best practices and standards to manage tigers and
encourages assessments to benchmark progress. It sets minimum standards for
the effective management of target species and encourages the assessment of
these standards. It is being implemented in seven tiger range countries. So,
statement 3 is correct.
So far a total of 23 tiger reserves of India have received CA|TS accreditation.
14 Tiger Reserves in India have received the accreditation of the CA|TS
in 2021: Manas, Kaziranga and Orang in Assam, Satpura, Kanha and Panna in
Madhya Pradesh, Pench in Maharashtra, Valmiki in Bihar, Dudhwa in Uttar
Pradesh, Sunderbans in West Bengal, Parambikulam in Kerala, Bandipur in
Karnataka and Mudumalai and Anamalai in Tamil Nadu.
3 tiger reserves Sathyamangalam in Tamil Nadu and Bandhavgarh and
Pench in Madhya Pradesh have been conferred with the Conservation
Assured Tiger Standards (CA|TS) in 2022.
6 tiger reserves namely Kali, Melghat, Navegaon, Nagzira, Pilibhit and
Periyar have been awarded with CA|TS accreditation in 2023. So, statement
4 is not correct.

So, only two of the above statements are correct.


Therefore, option (b) is the correct answer.

Knowledge Box

Project Tiger:

It was launched in 1973 by the Government of India. It aims to foster an


exclusive tiger agenda in the core areas of tiger reserves, with an inclusive
people-oriented agenda in the buffer zone.
It is an ongoing Centrally Sponsored Scheme of the Union Ministry of
Environment, Forests and Climate Change providing central assistance to the
tiger States for their conservation in designated tiger reserves.

Relevance: The National Tiger Conservation Authority (NTCA) recently approved


shifting three tigresses from Ranthambore Tiger Reserve (RTR) to two other State
reserves.

Question 25. + 2.0 - 0.66

The ‘Cut Rootstock Method’ is sometimes seen in the news for which one of the
following purposes?

A. Clearing off the agricultural field after harvest in a sustainable manner

B. Increasing water retention capacity in paddy cultivation


C. Eliminating the need for soil preparation in vertical farming

  D. Protecting plants from invasive weed species

Question Analytics
3484 USERS 899 USERS 2585 USERS 52.9 SECS

ATTEMPTED  SOLVED CORRECTLY  SOLVED INCORRECTLY  YOUR TIME

51.23 SECS 0.0 SECS

 AVG. SOLVING TIME  FASTEST SOLVING TIME

Explanation :

Cut Rootstock Method:

It is a scientific approach to controlling the spread of Lantana Camara, an


invasive weed species and to make way for native plants. The weed has spread
over a large part hence, obstructing the way for native plants.
The method attempts to make the growth of weed dormant through a hormonal
imbalance and ensures it doesn’t germinate. It is done by manually uprooting the
weed and cutting its roots underneath the ground by an edge of a few inches.
The cycle additionally includes holding the evacuated plant upside down to kill it
and removing the more modest lantana saplings, alongside the greater weed,
physically.
When the species is cleared, it permits the proliferation of the local bushes and
creepers, which have frequently been viewed as immature or are attempting to
develop normally close to the invasive species. This method of weeding does not
require any automated machine.

Therefore, option (d) is the correct answer.

Relevance: Recently, the Cut Rootstock Method was suggested to control the spread of
Lantana Camara in Asola Bhatti Wildlife Sanctuary.

Question 26. + 2.0 - 0.66

Which one of the following was recently declared as India’s first Dark Sky Park?

 A. Changthang Wildlife Sanctuary in Ladakh

B. Desert National Park in Rajasthan

C. Valley of Flowers National Park in Uttarakhand

 D. Pench Tiger Reserve in Maharashtra

Question Analytics
4078 USERS 1387 USERS 2691 USERS 11.9 SECS

ATTEMPTED  SOLVED CORRECTLY  SOLVED INCORRECTLY  YOUR TIME

23.8 SECS 0.0 SECS

 AVG. SOLVING TIME  FASTEST SOLVING TIME

Explanation :

The Pench Tiger Reserve (PTR) in Maharashtra has attained the distinction of
becoming India’s inaugural Dark Sky Park and the fifth in Asia.
The PTR, also known as Pench National Park, stands as one of India’s premier
Tiger Reserves and the first to span two States – Madhya Pradesh and
Maharashtra.
The decision to designate Pench as the first Dark Sky Park is driven by the
need to shield the night sky from light pollution.
India’s first dark-sky Reserve (not Park) is the Indian Astronomical
Observatory (IAO), a high-altitude astronomy station situated in Hanle village
(part of Changthang WLS) and operated by the Indian Institute of Astrophysics.
Positioned in the Western Himalayas at an elevation of 4,500 meters (14,764
ft), the IAO ranks among the world’s highest sites for optical, infrared and
gamma-ray telescopes.

Therefore, option (d) is the correct answer.

Knowledge Box

Dark Sky Park:

They are areas designated for the protection of night skies from light pollution,
resulting in an environment that allows for the local ecosystem to thrive.
These places are not only important for nature and wildlife but equally aid in
research for those interested in astronomy.
The use of artificial lights to provide visibility affects humans, animals and
hinders the ability to gaze into the celestial landscape with clarity.
A dark-sky preserve is an area, typically surrounding a park or observatory, that
limits artificial light pollution. The primary objective of the dark-sky movement is
generally to endorse astronomy.
Difference between Dark Sky Reserves and Dark Sky Park:
Reserves consist of a core area meeting minimum criteria for sky quality
and natural darkness, and a peripheral area that supports dark sky
preservation in the core. Reserves are formed through a partnership of
multiple land managers who have recognized the value of the natural
nighttime environment through regulations and long-term planning.
The land may be publicly or privately owned, provided that the
landowner(s) consent to the right of permanent, ongoing public access to
specific areas included in the DarkSky Park designation.

Relevance: Recently, Pench Tiger Reserve became India’s first Dark Sky Park.

Question 27. + 2.0 - 0.66

Which of the following are marine mammals?

1. Seals

2. Whales

3. Walruses
4. Penguins

5. Dugongs

Select the correct answer using the code given below:

A. 1, 2 and 4 only

B. 3, 4 and 5 only

  C. 1, 2, 3 and 5 only

D. 1, 2, 3, 4 and 5

Question Analytics
4343 USERS 2832 USERS 1511 USERS 34.7 SECS

ATTEMPTED  SOLVED CORRECTLY  SOLVED INCORRECTLY  YOUR TIME

46.02 SECS 0.0 SECS

 AVG. SOLVING TIME  FASTEST SOLVING TIME

Explanation :

Marine mammals are found in marine ecosystems around the globe. They are a
diverse group of mammals with unique physical adaptations that allow them to
thrive in the marine environment with extreme temperatures, depths,
pressure, and darkness.
Marine mammals are classified into four different taxonomic groups:
Cetaceans: whales, dolphins, and porpoises etc.
Pinnipeds: seals, sea lions and walruses etc.
Sirenians: manatees and dugongs etc.
Marine fissipeds: polar bears and sea otters etc.
So, seals, whales, walruses and dugongs are marine mammals. So, points
1, 2, 3 and 5 are correct.
Penguins are specialised marine birds (not marine mammals) adapted to living
at sea. So, point 4 is not correct.

Therefore, option (c) is the correct answer.

Relevance: According to the IUCN Red List human activity and climate change is
devastating marine species from mammals to corals.

Question 28. + 2.0 - 0.66

In India, hunting of a wild animal mentioned in Schedule I of the Wildlife Protection


Act, 1972, is permitted in which of the following circumstances?
1. If a wild animal is causing damage to property.

2. If a wild animal is disabled or diseased beyond recovery and cannot be captured.


3. If a person was acting in defense of oneself or another person.

Select the correct answer using the code given below:

A. 1 only

 B. 1 and 3 only

C. 2 only

 D. 2 and 3 only

Question Analytics
4310 USERS 2331 USERS 1979 USERS 20.6 SECS

ATTEMPTED  SOLVED CORRECTLY  SOLVED INCORRECTLY  YOUR TIME

56.77 SECS 0.0 SECS

 AVG. SOLVING TIME  FASTEST SOLVING TIME

Explanation :

Hunting of wild animals specified in Schedule I of the Wildlife Protection Act,


1972 can only be permitted when they become dangerous to human life or are
beyond recovery, not when causing damage to property. So, statement 1 is not
correct.
The Chief Wildlife Warden may permit the hunting of a wild animal specified in
Schedule I if it is disabled or diseased beyond recovery and cannot be captured,
tranquilised or translocated. So, statement 2 is correct.
While hunting is allowed in self-defense or in defense of another person, it
should be noted that this should not exonerate anyone who was committing an act
in contravention of the Wildlife Protection Act at the time when such defense was
necessary. So, statement 3 is correct.
The Chief Wildlife Warden can grant a permit for hunting wild animals for
education, scientific research, or scientific management purposes, provided
certain conditions are met.

Therefore, option (d) is the correct answer.

Relevance: The state of Kerala urged the Union Government to make some
amendments to the Wildlife Protection Act of 1972.

Question 29. + 2.0 - 0.66

Terracotta models of mummies were discovered at which one of the following Indus
Valley Civilization sites?

A. Rakhigarhi

B. Dholavira

 C. Banawali

 D. Lothal

Question Analytics
3308 USERS 1039 USERS 2269 USERS 42.6 SECS

ATTEMPTED  SOLVED CORRECTLY  SOLVED INCORRECTLY  YOUR TIME

31.04 SECS 0.0 SECS

 AVG. SOLVING TIME  FASTEST SOLVING TIME

Explanation :

Lothal is one of the prominent sites of the ancient Harappan Civilization, dating to
2600 BCE to 2000 BCE. It is located close to the Gulf of Khambhat.
The artefacts found in Lothal include models of boats, Persian Gulf seals, bangles,
a model of a terracotta mummy, a terracotta pyramid, a Sumerian head,
precious and semi-precious stones like carnelian and lapis lazuli.
Being a port town, Lothal hosted traders from different parts of the world and
civilizations. Along with trade products, their cultural beliefs and values, ideas and
innovative techniques also transferred to Lothal, making it the hub of the culture
and economy of the world.

Therefore, option (d) is the correct answer.

Question 30. + 2.0 - 0.66

Consider the following:

1. Establishment of the institution of gotra


2. Nuclearisation of families

3. Beginning of the usage of iron


4. Emergence of temple worship

How many of the above developments took place during the Later Vedic phase?

A. Only one

  B. Only two

C. Only three

D. All four

Question Analytics
3961 USERS 1639 USERS 2322 USERS 40.9 SECS

ATTEMPTED  SOLVED CORRECTLY  SOLVED INCORRECTLY  YOUR TIME

50.25 SECS 0.0 SECS

 AVG. SOLVING TIME  FASTEST SOLVING TIME

Explanation :

Later Vedic Phase (1000 BC to 500 BC):

The institution of gotra developed in this period. This means that people having a
common gotra descended from a common ancestor and no marriage between
the members of the same gotra could take place. Monogamous marriages were
preferred even though polygamy was frequent. Women had to stay with their
husbands at their place after marriage. So, point 1 is correct.
The family remained the basic unit of the Vedic society. However, its composition
changed. The later Vedic family became large enough to be called a joint family
with three or four generations living together. The rows of hearths discovered at
Atranjikhera and at Ahichchhatra (both in western Uttar Pradesh) show that
these were meant for communal feeding or for cooking the food of large families.
So, point 2 is not correct.
The main factor in the expansion of the Aryan culture during the later Vedic
period was the beginning of the use of iron around 1000 BC. The Rigvedic people
knew of a metal called ayas which was either copper or bronze. In the later Vedic
literature ayas were qualified with Shyama or Krishna meaning black to denote
iron. So, point 3 is correct.
During the entire Vedic phase, people did not construct temples nor did they
worship any statues. These features of Indian religion developed much later. So,
point 4 is not correct.
There was an increase in the frequency and number of the yajna which
generally ended with the sacrifices of a large number of animals. This was
probably the result of the growing importance of a class of Brahmanas and their
efforts to maintain their supremacy in the changing society. Some of the important
yajnas were - ashvamedha, vajapeya, rajasuya etc.

So, only two of the above developments took place during the Later Vedic phase.

Therefore, option (b) is the correct answer.

Question 31. + 2.0 - 0.66

Consider the following Ashokan inscriptions:


1. Laghman

2. Ahraura

3. Mansehra
4. Shahbazgarhi

How many of the above are not found in India?

A. Only one

B. Only two

 C. Only three

 D. All four

Question Analytics
1860 USERS 539 USERS 1321 USERS 29.9 SECS

ATTEMPTED  SOLVED CORRECTLY  SOLVED INCORRECTLY  YOUR TIME

25.59 SECS 0.0 SECS


 AVG. SOLVING TIME  FASTEST SOLVING TIME

Explanation :

Ashoka (268 B.C.E to 232 B.C.E.) issued a large number of edicts that carried his
message concerning the idea and practice of dhamma. The Ashokan inscriptions
were written in multiple languages. The northwestern part of the Mauryan
Empire (which included areas of modern‑day Pakistan and Afghanistan) yielded
one Greek, four Aramaic, one Greek‑Aramaic and one Aramaic‑Prakrit inscription.
An inscription is found in the Laghman District on the left (northern) bank of the
Kabul River, a little above Jalalabad, in modern Afghanistan. So, point 1 is
correct.
A minor inscription is found in the Ahraura, which is located in the Mirzapur
district of Uttar Pradesh. So, point 2 is not correct.
Mansehra and Shahbazgarhi inscriptions are located in Pakistan. They are sets
of Major Rock Edicts in the Prakrit language and Kharosthi script. So, points 3 and
4 are correct.
The fourteen major edicts recorded at Shahbazgarhi site present aspects of
Asoka’s dharma or righteous law. The edicts are located beside one of the
ancient trade routes connecting the Vale of Peshawar with the valley of Swat,
Dir and Chitral to the North and the great city of Taxila to the South East.

So, only three of the above inscriptions are not found in India.
Therefore, option (c) is the correct answer.

Question 32. + 2.0 - 0.66

The notion of Arhat in Jainism and Bodhisattva in Buddhism is similar in which one of
the following ways?

Both seek enlightenment for themselves alone, ignoring the well-being of


A.
others.

B. Both believe that there is only a single facet of ultimate truth.

  C. Both seek to break away from the cycle of death and rebirth.

D. Both do not believe that actions can help to accumulate good karma.

Question Analytics
4209 USERS 2819 USERS 1390 USERS 21.8 SECS

ATTEMPTED  SOLVED CORRECTLY  SOLVED INCORRECTLY  YOUR TIME

52.42 SECS 0.0 SECS

 AVG. SOLVING TIME  FASTEST SOLVING TIME

Explanation :

While Arhats in Jainism aim for their own enlightenment, Bodhisattvas in


Buddhism prioritise the well-being and enlightenment of all sentient beings before
achieving it themselves. While Arhats in Jainism adhere to a strict set of rules,
Bodhisattvas in Buddhism follow a more flexible set of guidelines known as the
"six paramitas".
Both Arhats in Jainism and Bodhisattvas in Buddhism seek liberation from the
cycle of death and rebirth.
While both Arhats and Bodhisattvas aim to accumulate positive karma through
their actions, their primary goal is not merely to accumulate good karma but to
attain liberation from the cycle of rebirth.
Anekantavada is the fundamental doctrine of Jainism that emphasises that
the ultimate truth and reality are complex and have multiple aspects. Hence,
there exists non-absolutism, which means no single, specific statement can
describe the nature of existence and the absolute truth.

Therefore, option (c) is the correct answer.

Knowledge Box

About Arhat:

In the Jain sūtras, the term is often used in a sense closer to that found in
Buddhist writings. Here the arhat is described as one who is free from desire,
hatred and delusion, who knows everything, and who is endowed with
miraculous powers.
While these characterizations are consistent with the Buddhist use of the term, it
should be noted that the Jains applied the word exclusively to the tīrthankaras
or revealers of religion, whereas in Buddhism arhatship is an ideal to be
attained by all serious religious strivers, especially monks and nuns.

Question 33. + 2.0 - 0.66

With reference to the Delhi Sultanate, consider the following statements:

1. Alauddin Khilji used taxes from khalisa land for payment of cash salaries to the
soldiers.
2. Muhammad bin Tughlaq extended Alauddin Khilji’s system of revenue collection
based on measurement of land.
3. Firoz Shah Tughlaq introduced the levy of house tax (ghari) and cattle tax (charai).

Which of the statements given above are correct?

 A. 1 and 2 only

B. 2 and 3 only
C. 1 and 3 only

D. 1, 2 and 3

Question Analytics
2859 USERS 1134 USERS 1725 USERS 42.5 SECS

ATTEMPTED  SOLVED CORRECTLY  SOLVED INCORRECTLY  YOUR TIME

42.96 SECS 0.0 SECS

 AVG. SOLVING TIME  FASTEST SOLVING TIME

Explanation :

The rulers who ruled substantial parts of North India between AD1200 to AD1526 were
termed Sultans and the period of their rule was the Delhi Sultanate. These rulers were
of Turkish and Afghan origin. During this period, five different dynasties ruled Delhi:
Mamluks (AD 1206–AD 1290) (popularly known as the slave dynasty), the Khiljis (AD
1290–AD 1320), the Tughlaqs (AD 1320–AD 1412), the Sayyids (AD 1412–AD 1451) and
the Lodis (AD 1451– AD 1526).

Alauddin Khilji attempted to increase the revenue collection by enhancing the


demand, introducing direct collection and cutting down the leakages to the
intermediaries. The revenue collectors were ordered to demand the revenue with
such rigour that the peasants should be forced to sell their produce immediately at
the side of the fields. At another place, Alauddin Khalji brought the Doab into
Khalisa and the tax (mahsul) from there was spent on paying the cash salaries
to the soldiers. So, statement 1 is correct.
Muhammad bin Tughlaq first extended Alauddin Khilji’s system of revenue
collection based on measurement of land to Gujarat, Malwa, Deccan, South India
and Bengal. There was an enormous increase in agrarian taxation. Some
additional new imposts (abwab) were levied. Of the other taxes, kharaj, charai and
ghari were more rigorously collected. He introduced the practice of giving
agricultural loans named sondhar for increasing the area under plough and for
digging wells for irrigation. A new ministry designated diwan-i amir-i kohi was
established to promote agriculture. He became the first Sultan to attempt to
formulate an agricultural policy for promoting agriculture. So, statement 2 is
correct.
Firuz Shah Tughlaq (1351-88) abolished agrarian cesses and forbade levying of
ghari (house tax) and charai (cattle tax). But he is reported to have imposed a
separate tax – jiziya – distinct from kharaj (land tax) on the peasants. He also
introduced an irrigation tax (haqq-i sharb) in Haryana where he dug up canals.
So, statement 3 is not correct.

Therefore, option (a) is the correct answer.

Knowledge Box

Under Firuz Shah, ‘water tax’ (haqq-i sharb) was taken from those cultivators
who irrigated their land from the water supplied by the canals constructed by the
state.
Khalisa: It referred to territories whose revenues were directly collected for the
Sultan’s own treasury. Its size seems to have expanded quite considerably under
Alauddin Khalji. However, it did not appear to consist of shifting territories
scattered throughout the country.

Question 34. + 2.0 - 0.66

Which one of the following best describes the terms ‘Theragatha’ and ‘Therigatha’?

A. Kirtanghar established by Sankardeva

B. Jaina temples in Southern India

C. Hymns composed by Alvar saints

  D. Collection of poems narrated by Buddhist monks and nuns

Question Analytics
3875 USERS 3093 USERS 782 USERS 30.3 SECS

ATTEMPTED  SOLVED CORRECTLY  SOLVED INCORRECTLY  YOUR TIME

28.98 SECS 0.0 SECS

 AVG. SOLVING TIME  FASTEST SOLVING TIME

Explanation :

Theragatha (Verses of Elder Monks) and Therigatha (Verses of the Elder Nuns)
are a collection of poems with verses which were narrated by the early members
of the Buddhist Sangha.
In Theravada Buddhism, the Sutta Pitaka is a compilation of discourses ascribed
to the Buddha.
Specifically, the Khuddaka Nikaya holds a unique position within this collection,
encompassing diverse literary genres such as verses on ethics (Dhammapada),
hymns by monks (Theragatha), hymns by nuns (Therigatha), and narrative tales
illustrating moral lessons (Jataka).
Therigatha is the first surviving poetry supposed to have been composed by
women in India. Hence, it is important for not only Buddhism but also gender
studies. The gathas of the Therigatha strongly support the view that women are
equal to men in terms of spiritual attainment.

Therefore, option (d) is the correct answer.

Question 35. + 2.0 - 0.66

The motto of the Supreme Court of India, 'yatodharmastato jayah' (Truth alone I
uphold) is taken from:

 A. Mundaka Upanishad

 B. Mahabharata

C. Ramayana

D. Yajurveda

Question Analytics
4067 USERS 2080 USERS 1987 USERS 14.3 SECS

ATTEMPTED  SOLVED CORRECTLY  SOLVED INCORRECTLY  YOUR TIME

31.47 SECS 0.0 SECS

 AVG. SOLVING TIME  FASTEST SOLVING TIME

Explanation :
The inscription on the Dharm Chakra logo of the Supreme Court in Sanskrit
“yatodharmastato jayah” means – Truth alone I uphold. It is also referred to as
the wheel of righteousness, encompassing truth, goodness and equity.
It is a sober reminder to those who judge the nation’s laws against the high
standards of the Constitution, of the high standards to which they too must hold
themselves.
The phrase comes from the Hindu epic Mahabharata verse 13.153.39.

Therefore, option (b) is the correct answer.

Question 36. + 2.0 - 0.66

Consider the following pairs:

Sl. No. Temple Architecture Style Example

1. Solanki : Somnath Temple at Kathiawar

2. Nagara : Kailashnath Temple at Ellora

3. Vesara : Doddabasappa Temple at Dambal

4. Dravidian : Chennakeshava Temple at Belur

How many of the above pairs are correctly matched?

A. Only one pair

 B. Only two pairs

C. Only three pairs

D. All four pairs

Question Analytics
2782 USERS 1473 USERS 1309 USERS 18.9 SECS

ATTEMPTED  SOLVED CORRECTLY  SOLVED INCORRECTLY  YOUR TIME

43.84 SECS 0.0 SECS

 AVG. SOLVING TIME  FASTEST SOLVING TIME

Explanation :

Solanki Style of Temple Architecture: It flourished in Gujarat. It consists of a


closed hall (‘sanctum’) and a porch that is inter-connected both internally and
externally. A detached peri-stylar hall is added in larger temples on the same axis,
which is often preceded by a ‘torana’ or ornamental arched entrance. These
temples were largely built in sandstone or limestone. The Sun Temple at
Modhera, the Vemala Temple at Mount Abu and the Somnath Temple at
Kathiawar are examples of the Solanki architecture. So, pair 1 is correctly
matched.
Nagara Style of Temple Architecture: It emerged in northern India in the 5th
century CE, during the late Gupta period. The plan of this style is four-sided. The
garbhagriha is a perfect square whereas the whole temple plan could be oblong.
The garbhagriha or sanctum houses the image or idol of the main deity. The
pathway leading to the garbhagriha is the mandapa. The shikhara is tapered at
the top. Garbhagriha is surrounded by a pradakshinapatha or a
circumambulatory path. It generally rests on a high platform. Examples of Nagara
Temples include the Khajuraho Temple. So, pair 2 is not correctly matched.
Vesara Style of Temple Architecture: It is also known as the Karnataka School
of Architecture. It was conceptualised under the later Chalukya rulers in the
mid-seventh century CE. It combined the features of both Nagara school and
Dravidian school and resulted in a hybridised style. Some of its features
emphasise vimana and mandapa, an open ambulatory passageway. The pillars,
doorways and ceilings were decorated with intricate carvings. Examples include
the Kailashnath temple in Ellora, Doddabasappa Temple at Dambal. So, pair 3
is correctly matched.

Dravidian Style of Temple Architecture: It flourished under the patronage of the


Chola rulers. It was a continuation of the previous Pallava architecture, with
some variations. Unlike the Nagara temples, the Dravidian temples were
surrounded by high boundary walls. The front wall had a high entrance gateway
known as the gopuram. The temple premise was laid out in the panchayatan
style with a principal temple and four subsidiary shrines.
In the Dravidian style, the spire is in the form of a stepped pyramid that rises
up linearly rather than curved. It is known as vimana. There is only one
vimana in the Dravidian architecture on top of the main temple. The
subsidiary shrines do not have vimanas, unlike in Nagara architecture. The
presence of a water tank inside the temple enclosure is a unique feature of
the Dravidian style. Examples include Brihadeswara temple at Tanjore
and Gangaikondacholapuram temple.
Hoysala Style of Temple Architecture: In the region of Karnataka (near Mysore),
the temples built under the Hoysala rulers developed a distinct style of their own
known as the Hoysala School of art. Unlike the crucified ground plan of the
Panchayatan style, the shrines led out in the shape of an intricately designed
star. This was known as the Stellate plan. Examples include Hoysaleswara
Temple at Halebidu, Chennakesava Temple at Belur, Chennakesava Temple at
Somanathapura. So, pair 4 is not correctly matched.

So, only two of the above pairs are correctly matched.


Therefore, option (b) is the correct answer.

Relevance: Recently, three Hoysala-era temples in Karnataka were listed in UNESCO’s


World Heritage List.

Question 37. + 2.0 - 0.66

In the context of the social reforms during British India, which one of the following
organisations was formed earliest?

 A. Satyashodhak Samaj

 B. Prarthana Samaj

C. Arya Samaj

D. Ramakrishna Mission

Question Analytics
4052 USERS 1923 USERS 2129 USERS 27.8 SECS

ATTEMPTED  SOLVED CORRECTLY  SOLVED INCORRECTLY  YOUR TIME

39.72 SECS 0.0 SECS

 AVG. SOLVING TIME  FASTEST SOLVING TIME


Explanation :

Prarthana Samaj: In 1867, Keshab Chandra Sen helped Atmaram Pandurang


found the Prarthana Samaj in Bombay. Earlier, the Brahmo ideas spread in
Maharashtra. A precursor of the Prarthana Samaj was the Paramahansa Sabha,
something like a secret society to spread liberal ideas and encourage the
breakdown of caste and communal barriers. The emphasis was on monotheism,
but on the whole, the samaj was more concerned with social reforms than with
religion.
Satyashodhak Samaj (Society of Seekers of Truth): Jyotiba Phule formed this in
1873 to educate society about caste prejudice and to free downtrodden lower-caste
people from the stigmas created by Brahmins. Its main objectives included social
service and the spread of education among women and lower caste people.
Arya Samaj: It was founded by Swami Dayanand Saraswati in 1875 in Bombay.
The important task of this Socio-Religious Reform Movement was reforming the
Hindu religion in North India. It promoted social reform. It made an effort to
improve women's conditions. It promoted social equality while combating
untouchability and the rigidities of the hereditary caste system.
Ramakrishna Mission: It was initiated by Swami Vivekananda in 1897 to spread
the universal message of Vedanta in the light of Sri Ramakrishna Paramahamsa
(He was a teacher of Swami Vivekanand). The main motto of this initiative is to
work for the alleviating of the poor and the downtrodden with the motto ‘for one’s
liberation and the good of the world’.

Therefore, option (b) is the correct answer.

Question 38. + 2.0 - 0.66

Which one of the following was not a provision of the Indian Independence Act of
1947?

 A. It abolished the office of Viceroy.

It deprived the Governor General of the power to assent a bill in the name of
 B.
British Crown.

C. It dissolved the existing Central Legislative Assembly.

D. It continued the services of civil servants appointed by the Secretary of State.

Question Analytics
4090 USERS 855 USERS 3235 USERS 62.7 SECS

ATTEMPTED  SOLVED CORRECTLY  SOLVED INCORRECTLY  YOUR TIME

55.06 SECS 0.0 SECS

 AVG. SOLVING TIME  FASTEST SOLVING TIME

Explanation :

On July 5, 1947, the British Parliament passed the Indian Independence Act which
was based on the Mountbatten Plan (June 3, 1947) and the Act got royal assent on
July 18, 1947. The Act was implemented on August 15, 1947.
Some of the key features of the act are given below:
It ended British rule in India and declared India as an independent and
sovereign state on August 15, 1947.
It provided for the partition of India and the creation of two independent
dominions of India and Pakistan with the right to secede from the British
Commonwealth.
Each dominion was to have a Governor-General to be responsible for the
effective operation of the Act. The Constituent Assembly of each new
dominion was to exercise the powers of the legislature of that dominion and
the existing Central Legislative Assembly and the Council of States were
to be automatically dissolved.
It abolished the office of Viceroy and provided, for each dominion, a
Governor General, who was to be appointed by the British King on the
advice of the dominion cabinet.
It deprived the British Monarch of his right to veto bills or ask for reservation
of certain bills for his approval. But, this right was reserved for the Governor
General. The Governor General would have full power to assent to any
bill in the name of His Majesty.
It abolished the office of the Secretary of State for India and transferred
its functions to the Secretary of State for Commonwealth Affairs.
It discontinued the appointment to civil services and reservation of posts by
the Secretary of State for India. The members of the civil services
appointed before August 15, 1947, would continue to enjoy all benefits
that they were entitled to till that time.

Therefore, option (b) is the answer.

Question 39. + 2.0 - 0.66


Consider the following statements:
Statement-I:
The modern day on the Earth is longer than a century ago.

Statement-II:

The Earth’s rotation is slowing down due to the tidal effect of the Moon.
Which one of the following is correct in respect of the above statements?

Both Statement-I and Statement-II are correct and Statement-II is the correct
 A.
explanation for Statement-I

Both Statement-I and Statement-II are correct and Statement-II is not the
B.
correct explanation for Statement-I

 C. Statement-I is correct but Statement-II is incorrect

D. Statement-I is incorrect but Statement-II is correct

Question Analytics
3482 USERS 1585 USERS 1897 USERS 30.0 SECS

ATTEMPTED  SOLVED CORRECTLY  SOLVED INCORRECTLY  YOUR TIME

44.56 SECS 0.0 SECS

 AVG. SOLVING TIME  FASTEST SOLVING TIME

Explanation :

Earth rotates on its own axis from West to East. As viewed from the North Star or
Pole Star the Earth turns anti-clockwise. The Earth’s axis of rotation meets its
surface at the North Pole, in the northern hemisphere. The South Pole is the other
point where the axis of rotation intersects its surface, in Antarctica. The rotation of
the Earth is responsible for the alternate periods of light and darkness that give us
day and night.
The Earth rotates once every twenty-four hours to the Sun and once every 23
hours 56 minutes and 4 seconds to the stars. Earth’s rotation is slightly slowing
with time, thus the day was shorter in the past, this is due to the tidal effect of
the Moon on the Earth’s rotation. The atomic clock shows that the modern day
is longer by about 1.7 milliseconds than a century ago, slowly increasing the
rate at which UTC (Coordinated Universal Time) is adjusted by leap seconds. So,
statements 1 and 2 are correct.

So, both Statement-I and Statement-II are correct and Statement-II is the correct
explanation for Statement-I.
Therefore, option (a) is the correct answer.

Question 40. + 2.0 - 0.66

The term ‘Big Crunch’ is sometimes seen in the news in the context of:

A. study of the Moon’s south pole

B. origin and expansion of the universe from a single point

 C. impact of solar flares on satellites revolving around the Earth

 D. collapse of the universe back into a singular point

Question Analytics
3371 USERS 2095 USERS 1276 USERS 20.0 SECS

ATTEMPTED  SOLVED CORRECTLY  SOLVED INCORRECTLY  YOUR TIME

32.95 SECS 0.0 SECS

 AVG. SOLVING TIME  FASTEST SOLVING TIME

Explanation :

The term 'Big Crunch' refers to a scenario in cosmology where the expansion of
the universe eventually slows down and reverses, leading to the collapse of
the universe back into a singular point. This is a contrasting concept to the
more commonly discussed idea of the universe's continuous expansion.
As per the standard expectation, the future expansion of space on cosmic scales is
exponentially rapid. But there is an unlikely, though not inconceivable, possibility
that at some untold date, space’s expansion will reverse and the universe will
shrink down to a “Big Crunch”. In some theories, the Big Crunch precedes another
Big Bang and the cycle continues interminably.

Therefore, option (d) is the correct answer.

Knowledge Box

Theories regarding the universe:

Quantum field theory offers a vision, suggesting that the Higgs field, which
permeates the universe, could undergo a sudden drop in energy configuration
due to quantum effects, leading to the abrupt cessation of the known universe.
There is also the possibility of the decay of protons, which could impact the
longevity of celestial bodies. While current theories suggest a proton's lifetime
exceeding 10^73 years, some models propose decay as early as 10^35 years. In
such a scenario, white dwarfs and neutron stars might persist until 10^39 years
before disappearing into decay products.
The fate of the universe is also influenced by the unknown nature of dark matter,
adding an element of uncertainty to its ultimate trajectory.

Relevance: Recently, a physicist traced the universe’s past and future, from a raucous
infancy to a black-hole-dominated aeon of darkness.

Question 41. + 2.0 - 0.66

Which one of the following ecological corridors connects the Eastern and Western
Ghats?

A. Javadi Hills

B. Sahyadri Hills

 C. Biligiriranga Hills

 D. Nallamala Hills

Question Analytics
3979 USERS 1515 USERS 2464 USERS 9.7 SECS

ATTEMPTED  SOLVED CORRECTLY  SOLVED INCORRECTLY  YOUR TIME

37.18 SECS 0.0 SECS

 AVG. SOLVING TIME  FASTEST SOLVING TIME

Explanation :

The Eastern Ghats are a discontinuous range of mountains along India's eastern coast.
They run from northern Odisha through Andhra Pradesh to Tamil Nadu in the south
passing some parts of Karnataka. Western Ghats lie parallel to the western coast. They
are continuous and can be crossed through passes only. The Western Ghats are
higher than the Eastern Ghats. Their average elevation is 900– 1600 metres as against
600 metres of the Eastern Ghats.

The Biligiriranga Hills, which run east from the Western Ghats to the River
Kaveri, form a forested ecological corridor that connects the Eastern and
Western Ghats. This allows the second-largest wild Asian elephant population in
India to range between the South Eastern Ghats, the Biligiriranga Hills, the Nilgiri
Hills and the South Western Ghats.
The Eastern Ghats are older than the Western Ghats and have a complex
geologic history related to the assembly and breakup of the ancient
supercontinent of Rodinia and the assembly of the Gondwana supercontinent.
The structure of the Eastern Ghats includes thrusts and strike-slip faults all
along its range. They are made up of charnockites, granite gneisses, khondalites,
metamorphic gneisses and quartzite rock formations.

Therefore, option (c) is the correct answer.


Relevance: Recently, a National conference on ‘Tribal Heritage of Eastern Ghats’ was
organized in Visakhapatnam.

Question 42. + 2.0 - 0.66

Consider the following:

1. Sugarcane and Onion


2. Maize and Pigeon pea
3. Cotton and Barley

4. Groundnut and Pigeon pea


How many of the above crops are grown together using intercropping methods in
India?

A. Only one

B. Only two

 C. Only three

 D. All four

Question Analytics
2704 USERS 761 USERS 1943 USERS 27.7 SECS

ATTEMPTED  SOLVED CORRECTLY  SOLVED INCORRECTLY  YOUR TIME

37.44 SECS 0.0 SECS

 AVG. SOLVING TIME  FASTEST SOLVING TIME

Explanation :

Intercropping: It is a cropping system in which intercrop is grown in between rows of


the major crop, without affecting the optimum plant population as well as the yield of
the major crop (base crop). In this system, two or more crops are grown
simultaneously on the same piece of land with a distinct row arrangement.

Onion crop is best suited for intercrop with paired row planting of sugarcane
(November -December planting) under a drip irrigation system. It saves 25-30%
water. So, point 1 is correct.
Row cropping is practiced for maize and pigeon pea which increases their yield.
So, point 2 is correct.
Cotton being a Kharif crop is not intercropped with barley, a rabi crop. So, point
3 is not correct.
Double-up legume is a method of intercropping two legumes (in this case
groundnut and pigeon pea) that have different growth habits and take advantage
of beneficial interactions of the two legumes on the same piece of land increasing
their productivity. Pigeon peas are suitable for intercropping with groundnut
because of their slow growth in the first two months and only start rapid growth
when groundnut approaches maturity. So, point 4 is correct.
Cultivating black pepper, nutmeg and cinnamon in the coconut garden increases
crop production and provides additional income.

So, only three of the above crops are grown together using intercropping methods
in India.
Therefore, option (c) is the correct answer.

Knowledge Box

Benefits of intercropping include:


Better utilisation of growth resources like light, nutrients and moisture
Serves as insurance against failure of any one of the component crops, thus
providing an economy of space and time
Improves soil fertility, reduces soil crust formation and controls soil erosion
Control of pests and diseases and suppression of weeds
Cultivation practices for main crops supplement the requirement of
companion crops giving additional yield from the unit area
Care should be taken to select crops with different growth habits, root growth,
duration and families. The following points are to be considered while selecting
crops for intercropping:
Legume crops with non-legume crops
Crops selected should be of different families to avoid pests and diseases
Short-duration crops with long-duration crops
Tall crops with short ones and busy crops with erect ones

Relevance: Recently, Nekram Sharma, a Himachal farmer was awarded Padma Shri for
reviving the traditional ‘nau-anaj’ intercropping method.
Question 43. + 2.0 - 0.66

Consider the following:

1. Subtropical High Pressure Belt


2. Subpolar Low Pressure Belt

3. Equatorial Low Pressure Belt


4. Polar High Pressure Belt

How many of the above are primarily thermally induced pressure belts?

A. Only one

  B. Only two

C. Only three

D. All four

Question Analytics
3941 USERS 3030 USERS 911 USERS 42.4 SECS

ATTEMPTED  SOLVED CORRECTLY  SOLVED INCORRECTLY  YOUR TIME

45.23 SECS 0.0 SECS

 AVG. SOLVING TIME  FASTEST SOLVING TIME

Explanation :

Thermally-induced pressure belts are primarily formed due to incoming solar


radiation. These include the Equatorial Low Pressure Belt and Polar High
Pressure Belt. So, points 3 and 4 are correct.
While on the other hand, dynamically-induced pressure belts are primarily
formed due to the rotation of Earth. These include the Subtropical High Pressure
Belt and Subpolar Low Pressure Belt. So, points 1 and 2 are not correct.
Equatorial Low Pressure Belt:
This pressure belt is located in the vicinity of the equator between 10° N
and 10° S latitudes. The average pressure in this belt is less than 1013
millibars throughout. As these regions receive higher angled rays of the
sun throughout the year, temperatures are high. So, the ground gets
heated and the air near the earth’s surface gets warmed by out
radiation. As warm air rises, it comes under lower pressure and
expands. The rising air creates a lower pressure on the Earth’s surface.
Doldrums are associated with this belt.
Polar High Pressure Belts or Polar Highs:
The polar regions in both hemispheres are characterised by extremely
low temperatures throughout the year. Hence high pressure persists at
the poles in these regions. Winds from these belts blow towards sub-
polar low-pressure belts in both hemispheres.
Subtropical High Pressure Belt:
They are located between 25° to 35° latitudes in both hemispheres. The
most important feature of this pressure belt is that it is broken into a
number of high-pressure cells.
Subpolar Low Pressure Belts:
They are located between 60° to 70° latitudes in both hemispheres.
These are also primarily dynamically induced in spite of the fact that
these regions experience low temperatures throughout the year.

So, only two of the above are primarily thermally induced pressure belts.
Therefore, option (b) is the correct answer.

Question 44. + 2.0 - 0.66

Consider the following pairs:

SI. No. Famous Place River

1. Gaya : Phalgu

2. Ujjain : Shipra

3. Nashik : Krishna

4. Madurai : Cauvery

How many pairs given above are correctly matched?

A. Only one pair

 B. Only two pairs

C. Only three pairs

 D. All four pairs

Question Analytics
3168 USERS 1606 USERS 1562 USERS 16.1 SECS

ATTEMPTED  SOLVED CORRECTLY  SOLVED INCORRECTLY  YOUR TIME

36.37 SECS 0.0 SECS

 AVG. SOLVING TIME  FASTEST SOLVING TIME


Explanation :

Gaya in Bihar lies along the Phalgu River, a tributary of the Ganga. It is situated
near the junction of the Gangetic Plain and the Chota Nagpur plateau. The town of
Bodh Gaya, south of Gaya, is famous as the site of the Buddha’s enlightenment.
So, pair 1 is correctly matched.
The historical city of Ujjain is located on the river Shipra, which is a tributary of
the Chambal. This city has a Jyotirling, one city out of seven salvation-providing
cities, Gadhkalika and Harsiddhi, the two Shakti Peeths, and sacred Kumbh that
takes place in four cities of India. The cave of King Bhartari is found here. So,
pair 2 is correctly matched.
Nashik is an ancient city in the northwest region of Maharashtra. It is situated on
the banks of the Godavari River. Nashik is best known for being one of the
Hindu pilgrimage sites, the Kumbh Mela which is held every 12 years. It is called
the ‘Wine Capital of India’ as half of India’s vineyards and wineries are located in
Nashik. So, pair 3 is not correctly matched.
Madurai in Tamil Nadu is located on the Vaigai River. The compact old part of the
city—enclosed by the Anai, Naga and Pasu (Elephant, Snake and Cow) hills—is
centred on Meenakshi Amman (Minakshi-Sundareshwara) Temple. So, pair 4
is not correctly matched.

So, only two of the above pairs are correctly matched.

Therefore, option (b) is the correct answer.

Question 45. + 2.0 - 0.66

Consider the following:

1. Gulf of Aden
2. Iberian Peninsula
3. Persian Gulf

4. Sinai Peninsula

How many of the above surround the Red Sea?

A. Only one

  B. Only two

C. Only three

D. All four

Question Analytics
3773 USERS 2481 USERS 1292 USERS 33.2 SECS

ATTEMPTED  SOLVED CORRECTLY  SOLVED INCORRECTLY  YOUR TIME

45.12 SECS 0.0 SECS

 AVG. SOLVING TIME  FASTEST SOLVING TIME

Explanation :

The Red Sea is the world’s northernmost tropical sea and is also one of the most
heavily travelled waterways.
It is connected to the Arabian Sea and the Indian Ocean to the south through the
Gulf of Aden and the narrow strait of Bab-el-Mandeb. The northern portion of
the Red Sea is bifurcated by the Sinai Peninsula into the Gulf of Aqaba and the
Gulf of Suez, where it is connected to the Mediterranean Sea via the Suez Canal.
So, points 1 and 4 are correct.
The Persian Gulf does not surround the Red Sea. So, point 3 is not correct.
A total of 6 countries of Asia and Africa border the Red Sea. It is bordered by
Yemen and Saudi Arabia to the east, Egypt to the north and west and Sudan,
Eritrea and Djibouti to the west.
The Iberian Peninsula is located on Europe’s southwestern tip. It is part of the
southern Europe peninsula, which comprises three peninsulas; Iberian, Balkan,
and Italian peninsulas. The Iberian is the westernmost peninsula of the three
peninsulas. Its southern tip is separated from the African continent by the
narrow Strait of Gibraltar. At the narrowest point, the peninsula is only 8
kilometers from Africa, specifically Morocco. So, point 2 is not correct.

So, only two of the above surround the Red Sea.


Therefore, option (b) is the correct answer.

Relevance: Militant attacks in the Red Sea and surging freight rates are causing a split
in the global oil market.

Question 46. + 2.0 - 0.66

Which of the following countries’ spacecraft have made a landing on the Moon?
1. Japan
2. United Kingdom

3. China
4. France
5. South Korea
Select the correct answer using the code given below:

  A. 1 and 3 only

B. 1, 2 and 5 only

C. 2, 3 and 4 only

D. 2, 3, 4 and 5 only

Question Analytics
4297 USERS 3479 USERS 818 USERS 36.9 SECS

ATTEMPTED  SOLVED CORRECTLY  SOLVED INCORRECTLY  YOUR TIME

44.05 SECS 0.0 SECS

 AVG. SOLVING TIME  FASTEST SOLVING TIME

Explanation :

Only a few countries- the United States of America (USA), Russia (formerly
the USSR), China, India, and Japan have successfully landed on the Moon using
an impactor or a lander. A lander is a spacecraft designed to land on a celestial
body while an impactor gets destroyed after landing.
In 2024, Japan's Smart Lander for Investigating Moon (SLIM) became the fifth
country to soft-land on the Moon. So, point 1 is correct.
China has successfully landed on the Moon twice, in 2013 and 2019. Chang'e 4 was
launched in 2018 and landed on the moon in 2019. It was the first probe to land
on the Moon's far side. So, point 3 is correct.
The United Kingdom, France and South Korea have not sent a spacecraft to the
Moon. So, points 2, 4 and 5 are not correct.
Recently, India made history by becoming the first country to touch down near
the little-explored South Pole region and join the USA, Russia (Soviet Union) and
China in achieving a moon landing.

Therefore, option (a) is the correct answer.

Question 47. + 2.0 - 0.66

Consider the following statements:


1. Quantum dots are fluorescent nanocrystals with a semiconductor material at its core.
2. Smaller quantum dots emit high-frequency photons with shorter wavelengths.

Which of the statements given above is/are correct?


A. 1 only

B. 2 only

 C. Both 1 and 2

D. Neither 1 nor 2

Question Analytics
3366 USERS 2654 USERS 712 USERS 15.4 SECS

ATTEMPTED  SOLVED CORRECTLY  SOLVED INCORRECTLY  YOUR TIME

32.12 SECS 0.0 SECS

 AVG. SOLVING TIME  FASTEST SOLVING TIME

Explanation :

The Nobel Prize in Chemistry 2023 rewards the discovery and development of
Quantum Dots (QDs), nanoparticles so tiny that their size determines their
properties.
QDs are fluorescent semiconductor nanocrystals, composed of materials from
the elements in the periodic groups of II–VI, III–V or IV–VI, e.g. admium telluride
(Cd from group II and Te from group VI) and indium phosphamide (In from group
III and P from group V). They range in size from 2 to 10 nm in diameter and
contain approximately 200–10,000 atoms. So, statement 1 is correct.
QDs emit light and the size of nanocrystal determines the wavelength of light
emitted by it. A smaller quantum dot emits high-frequency photons with
shorter wavelengths. So, statement 2 is correct.
When particles are just a few nanometres in diameter, the space available to
electrons shrinks. This affects the particle’s optical properties. The luminous
properties of quantum dots are utilised in computer and television screens
based on QLED technology, where the Q stands for quantum dot.

Therefore, option (c) is the correct answer.

Relevance: The Nobel Prize in Chemistry 2023 was awarded to Moungi G. Bawendi,
Louis E. Brus and Aleksey Yekimov "for the discovery and synthesis of quantum dots".

Question 48. + 2.0 - 0.66

Consider the following:

1. Telecom service providers


2. Internet service providers
3. Search engines

4. Online payment sites


5. Cyber cafes

How many of the above are considered as ‘Intermediaries’ under the Information
Technology Act, 2000?

A. Only two

B. Only three

C. Only four

  D. All five

Question Analytics
3639 USERS 1027 USERS 2612 USERS 21.0 SECS

ATTEMPTED  SOLVED CORRECTLY  SOLVED INCORRECTLY  YOUR TIME

40.92 SECS 0.0 SECS

 AVG. SOLVING TIME  FASTEST SOLVING TIME

Explanation :

Information Technology Act, 2000 is the primary law in India dealing with
cybercrime and electronic commerce. It defines an intermediary as a person who
receives, stores or transmits any electronic record and provides any service relating
to such record on the behalf of another person.
Intermediaries include network service providers, telecom service
providers, internet service providers, search engines, web-hosting service
providers, online-auction sites, online payment sites, online-marketplaces
and cyber cafes. So, points 1, 2, 3, 4 and 5 are correct.
Intermediaries perform functions such as hosting content, collecting information
and evaluating information, facilitating communication and information
exchange, facilitating the use of the internet etc. Examples of intermediaries
include social media platforms such as WhatsApp, Twitter, Instagram and
Facebook; e-commerce sites such as Myntra and Amazon; search engines, cloud
service providers etc.

So, all five of the above are considered as ‘Intermediaries’ under the Information
Technology Act, 2000.
Therefore, option (d) is the correct answer.
Question 49. + 2.0 - 0.66

Consider the following statements:

1. Artificial General Intelligence (AGI) is a type of artificial intelligence that is limited to


a specific or narrow area.

2. Natural Language Processing (NLP) enables computers to analyse and understand


human language, both written and spoken.
Which of the statements given above is/are correct?

A. 1 only

  B. 2 only

C. Both 1 and 2

D. Neither 1 nor 2

Question Analytics
3703 USERS 2482 USERS 1221 USERS 33.4 SECS

ATTEMPTED  SOLVED CORRECTLY  SOLVED INCORRECTLY  YOUR TIME

34.29 SECS 0.0 SECS

 AVG. SOLVING TIME  FASTEST SOLVING TIME

Explanation :

Artificial General Intelligence (AGI), refers to a proposed type of artificial


intelligence that has the ability to understand, learn and apply its intelligence to a
wide range of problems, much like a human being.
Weak Artificial Intelligence (AI) — also called narrow AI — is a type of
artificial intelligence that is limited to a specific or narrow area.
Unlike narrow or weak AI, which is designed to perform specific tasks (like image
recognition, language translation, or playing chess), AGI can theoretically
perform any intellectual task that a human being can and more. Statement 1
is not correct.
Natural Language Processing (NLP) is a field of Artificial Intelligence (AI) that
enables computers to analyze and understand human language, both written
and spoken. It was formulated to build software that generates and comprehends
natural languages so that a user can have natural conversations with a computer
instead of through programming or artificial languages like Java or C. So,
statement 2 is correct.
NLP combines computational linguistics—rule-based modeling of human
language—with statistical and machine learning models to enable computers
and digital devices to recognize, understand and generate text and speech.

Therefore, option (b) is the correct answer.

Relevance: The Competition Commission of India will soon initiate a market study on
Artificial Intelligence (AI) to develop a comprehensive understanding of its evolving
landscape and use cases in markets in India.

Question 50. + 2.0 - 0.66

Consider the following:

1. Maglev trains
2. Experimental nuclear fusion reactors

3. Cell phone base stations


4. Particle detectors
Applications of superconductivity can be used in how many of the above fields?

A. Only one

B. Only two

C. Only three

  D. All four

Question Analytics
3362 USERS 2496 USERS 866 USERS 15.9 SECS

ATTEMPTED  SOLVED CORRECTLY  SOLVED INCORRECTLY  YOUR TIME

28.69 SECS 0.0 SECS

 AVG. SOLVING TIME  FASTEST SOLVING TIME

Explanation :

A superconductor is a material that achieves superconductivity, which is a state of


matter that has no electrical resistance and does not allow magnetic fields to
penetrate. An electric current in a superconductor can persist indefinitely.
Superconductivity can only typically be achieved at very cold temperatures. The credit
for the discovery of superconductivity goes to Dutch physicist Heike Kamerlingh
Onnes.
In order to generate the strong magnetic fields used in Magnetic Resonance
Imaging (MRI) and Nuclear Magnetic Resonance Imaging (NMRI), the machines
use powerful electromagnets.
Similar superconducting electromagnets are also used in maglev trains,
experimental nuclear fusion reactors and high-energy particle accelerator
laboratories. Superconductors are also used to power railguns and coilguns, cell
phone base stations, fast digital circuits and particle detectors. So, points 1, 2,
3 and 4 are correct.

So, all four of the above are the applications of superconductivity.

Therefore, option (d) is the correct answer.


Relevance: The scientific community now firmly believes that LK-99 is not a
superconductor operating at room temperature.

Question 51. + 2.0 - 0.66

Consider the following:


1. NITI Aayog

2. Cabinet Secretariat
3. National Disaster Management Authority
4. Central Bureau of Investigation

How many of the above are neither Constitutional nor statutory bodies/institutions in
India?

A. Only one

B. Only two

  C. Only three

D. All four

Question Analytics
4405 USERS 1334 USERS 3071 USERS 41.1 SECS

ATTEMPTED  SOLVED CORRECTLY  SOLVED INCORRECTLY  YOUR TIME

45.27 SECS 0.0 SECS

 AVG. SOLVING TIME  FASTEST SOLVING TIME

Explanation :
NITI Aayog: It was formed via a resolution of the Union Cabinet on 1 January
2015. It is chaired by the Prime Minister. The Governing Council of NITI Aayog,
comprising Chief Ministers of all the States and Union Territories with legislatures
and Lt Governors of other Union Territories, came into effect on 16 February 2015
via a notification by the Cabinet Secretariat. So, point 1 is correct.
Cabinet Secretariat: It is responsible for the administration of the Government
of India (Transaction of Business) Rules, 1961 and the Government of India
(Allocation of Business) Rules 1961, facilitating smooth transaction of business
in Ministries/ Departments of the Government. This Secretariat provides
Secretarial assistance to the Cabinet and its Committees, and also assists in
decision-making in Government. It is neither a Constitutional nor statutory
institution. So, point 2 is correct.
National Disaster Management Authority (NDMA): It is established under the
Disaster Management Act, 2005 to spearhead and implement a holistic and
integrated approach to disaster management in the country. Initially, the NDMA
was constituted in 2005 by an Executive Order of the Government of India.
Subsequently, the NDMA was notified in 2006 under the provisions of the Act. So,
point 3 is not correct.
Central Bureau of Investigation: It was set up in 1963 by a resolution of the
Ministry of Home Affairs. Later, it was transferred to the Ministry of Personnel,
Public Grievances and Pensions and now it enjoys the status of an attached office.
The Special Police Establishment (which looked into vigilance cases) setup in 1941
was also merged with the CBI. So, point 4 is correct.

So, only three of the above are neither Constitutional nor statutory
bodies/institutions in India.
Therefore, option (c) is the correct answer.

Question 52. + 2.0 - 0.66

Consider the following statements with reference to the Joint Sitting of the Parliament:
1. It is obligatory for the President to summon the Houses to meet in a joint sitting when
there is a deadlock over the passage of a Bill.
2. Once the President has notified his intention to summon the Houses for a joint sitting,
dissolution of Lok Sabha does not affect the proceeding with the Bill.
Which of the statements given above is/are correct?

A. 1 only
  B. 2 only

C. Both 1 and 2

D. Neither 1 nor 2

Question Analytics
4269 USERS 1281 USERS 2988 USERS 26.4 SECS

ATTEMPTED  SOLVED CORRECTLY  SOLVED INCORRECTLY  YOUR TIME

50.34 SECS 0.0 SECS

 AVG. SOLVING TIME  FASTEST SOLVING TIME

Explanation :

Joint sitting of the Parliament is an extraordinary machinery provided by the


Constitution of India to resolve a deadlock between the two Houses over the passage of
a Bill.

This provision is only an enabling one, empowering the President to take a step
for resolving a deadlock between the two Houses. It is not obligatory upon him
to summon the Houses to meet in a joint sitting. Moreover, this provision does
not disable the receiving House from passing the Bill after the lapse of six months,
provided the Bill has not lapsed by reason of dissolution or the President has not
already notified his intention to convene a joint sitting. So, statement 1 is not
correct.
A Bill upon which the Houses have disagreed and the President has notified his
intention to summon a Joint Sitting of the Houses to consider the Bill prior to
dissolution does not lapse on the dissolution of the Lok Sabha.
Once the President has notified his intention to summon the Houses for a
joint sitting, dissolution of Lok Sabha does not stand in the way of proceeding
with the Bill at a joint sitting. So, statement 2 is correct.
The Secretary General, Lok Sabha, issues summons to each member of Lok Sabha
and Rajya Sabha, specifying the time and place fixed by the President for the joint
sitting. At a joint sitting, the Speaker of Lok Sabha presides and the Secretary-
General, Lok Sabha acts as Secretary-General of the joint sitting.

Therefore, option (b) is the correct answer.


Relevance: President of India addressed a joint sitting of Lok Sabha and Rajya Sabha to
mark the start of the Budget Session of Parliament.

Question 53. + 2.0 - 0.66


Consider the following statements with reference to the Indian Judiciary:
1. A Supreme Court judge can be removed only on the ground of violation of the
Constitution.

2. A retired Supreme Court Judge cannot take up post-retirement appointments from


the Government of India.
3. A retired High Court Judge cannot plead in any High Court within the territory of
India.
How many of the above statements are correct?

 A. Only one

B. Only two

C. All three

 D. None

Question Analytics
4321 USERS 1201 USERS 3120 USERS 43.5 SECS

ATTEMPTED  SOLVED CORRECTLY  SOLVED INCORRECTLY  YOUR TIME

58.52 SECS 0.0 SECS

 AVG. SOLVING TIME  FASTEST SOLVING TIME

Explanation :

A Judge of the Supreme Court can be removed from his Office by an order of the
President. The President can issue the removal order only after an address by
Parliament has been presented to him in the same session for such removal. The
address must be supported by a special majority of each House of Parliament. The
grounds of removal are two: proved misbehaviour or incapacity. So,
statement 1 is not correct.
Article 124 (7) of the Constitution of India provides that the retired judges of the
Supreme Court are prohibited from pleading or acting in any Court or before any
authority within the territory of India. However, at present, there is no law that
prevents a retired judge of the Supreme Court or High Courts from taking up a
post-retirement appointment from the Government of India. So, statement 2 is
not correct.
The retired permanent judges of a High Court are prohibited from pleading or
acting in any court or before any authority in India except the Supreme Court
and the other High Courts. So, statement 3 is not correct.
So, none of the above statements are correct.

Therefore, option (d) is the correct answer.


Relevance: Recently, the Supreme Court of India dismissed a public interest litigation
(PIL) petition seeking a ‘cooling off’ period of two years before any retired judge of a
constitutional court can accept a post-retirement appointment.

Question 54. + 2.0 - 0.66

Which of the following form the Selection Committee to appoint members of the
Election Commission of India?

A. Prime Minister, Leader of Opposition in Lok Sabha and Chief Justice of India

B. Prime Minister, Chief Justice of India and Union Cabinet Minister

Prime Minister, Leader of Opposition in Lok Sabha and Union Cabinet


  C.
Minister

Prime Minister, Leader of Opposition in Lok Sabha and Leader of Opposition


D.
in Rajya Sabha
Question Analytics
4545 USERS 3491 USERS 1054 USERS 39.7 SECS

ATTEMPTED  SOLVED CORRECTLY  SOLVED INCORRECTLY  YOUR TIME

31.66 SECS 0.0 SECS

 AVG. SOLVING TIME  FASTEST SOLVING TIME

Explanation :

The Chief Election Commissioner and Other Election Commissioners


(Appointment, Conditions of Service and Term of Office) Act, 2023 governs the
procedure for appointment of Chief Election Commissioner and Election
Commissioners of the Election Commission of India.
The Chief Election Commissioner and other Election Commissioners shall be
appointed by the President by warrant under his hand and seal. A Search
Committee headed by the Minister of Law and Justice and comprising two other
members not below the rank of Secretary to the Government of India shall
prepare a panel of five persons for consideration of the Selection Committee.
The Chief Election Commissioner and other Election Commissioners shall be
appointed by the President on the recommendation of a Selection Committee
consisting of:
Prime Minister—Chairperson
Leader of Opposition in the Lok Sabha or Member, where the Leader of
Opposition in the Lok Sabha has not been recognised as such, the leader of
the single largest party in opposition of the Government in the House of the
People shall be deemed to be the Leader of Opposition
Union Cabinet Minister to be nominated by the Prime Minister.
Before this Act was passed, the Supreme Court in 2023 ruled that the
appointment of the Chief Election Commissioner and the Election Commissioners
shall be made by the President on the advice of a Committee consisting of the
Prime Minister, the Leader of the Opposition of the Lok Sabha, and in case no
Leader of the Opposition is available, the leader of the largest opposition Party in
the Lok Sabha in terms of numerical strength, and the Chief Justice of India.
However, the Court was careful to specify that these norms were “subject to any
law to be made by Parliament’. Parliament accordingly passed the Chief Election
Commissioner and Other Election Commissioners (Appointment, Conditions of
Service and Term of Office) Act, 2023.

Therefore, option (c) is the correct answer.


Relevance: Recently, Parliament has passed the 'Chief Election Commissioner and other
Election Commissioners (Appointment, Conditions of Service and Term of Office) Bill,
2023'.

Question 55. + 2.0 - 0.66

Consider the following statements:


1. A summon is issued to call a person to appear before the court whereas a warrant
empowers law enforcement officials to take specific actions.

2. A summon can be issued by an administrative authority whereas a warrant is a legal


document issued by a judicial officer.

Which of the statements given above is/are correct?

A. 1 only

B. 2 only

  C. Both 1 and 2

D. Neither 1 nor 2

Question Analytics
4227 USERS 3084 USERS 1143 USERS 41.6 SECS

ATTEMPTED  SOLVED CORRECTLY  SOLVED INCORRECTLY  YOUR TIME


41.68 SECS 0.0 SECS

 AVG. SOLVING TIME  FASTEST SOLVING TIME

Explanation :

A summon is a formal legal notice issued by a court or a judicial officer to call a


person to appear before the court in both criminal and civil cases. It serves as a
means to notify the accused (in criminal cases) or the defendant (in civil cases)
about the charges or claims brought against them. On the other hand, a warrant is
a formal legal document issued by a court or a judicial authority that empowers
law enforcement officials to take specific actions. So, statement 1 is correct.
In the context of criminal cases, a warrant can be an “arrest warrant,” which
authorises the police to apprehend a specific individual accused of
committing a crime. Alternatively, it can be a “search warrant,” granting the
authorities permission to conduct a search at a specified location to gather
evidence related to a crime.
A summons is also a legal document that notifies an individual or entity that they
are required to appear in court or respond to a legal proceeding. It is issued by a
court or administrative agency and typically informs the recipient of the date,
time, and location they need to appear.
A warrant is a document issued by a Court or judicial officer to a person or
an entity involved in a legal proceeding. A warrant is issued only in serious
offences and/or after duly served summons is disobeyed or if the accused has
wilfully avoided the services of the summons. So, statement 2 is correct.

Therefore, option (c) is the correct answer.

Question 56. + 2.0 - 0.66

Consider the following:


1. Holders of Bharat Ratna decoration
2. Cabinet Secretary
3. Judges of High Courts

4. Deputy Chief Ministers of States


5. Ambassadors of foreign countries
How many of the above are exempted from security checks at the airports in India?

 A. Only two

B. Only three
 C. Only four

D. All five

Question Analytics
3046 USERS 599 USERS 2447 USERS 25.9 SECS

ATTEMPTED  SOLVED CORRECTLY  SOLVED INCORRECTLY  YOUR TIME

47.18 SECS 0.0 SECS

 AVG. SOLVING TIME  FASTEST SOLVING TIME

Explanation :

The Ministry of Civil Aviation gives exemption from the pre-embarkation


security checks in consultation with or after receiving inputs from the Ministry of
Home Affairs (MHA). The list of VVIPs/VIPs exempt from pre-embarkation security
check is as follows:
President, Vice President, Prime Minister, Governors of States, Former
Presidents, Former Vice-President, Chief Justice of India, Speaker of Lok
Sabha, Union Ministers of Cabinet rank, Chief Ministers of States, Deputy
Chief Ministers of States, NITI Aayog, Leader of Opposition in Lok Sabha &
Rajya Sabha, Holders of Bharat Ratna Decoration, Ambassadors of
foreign countries, Charge D` Affairs and High Commissioners and their
spouses, Judges of Supreme Court, Chief Election Commissioner, Comptroller
& Auditor General of India, Deputy Chairman of Rajya Sabha & Deputy
Speaker of Lok Sabha, Minister of State of the Union Council of Ministers,
Attorney General of India, Cabinet Secretary, Lt. Governors of Union
Territories, Chiefs of staffs holding the rank of full General or equivalent
rank, Chief Justices of the High Courts, Chief Ministers of Union Territories,
Deputy Chief Ministers of Union Territories, Visiting Foreign dignitaries of
the above status, H.H. the Dalai Lama. So, points 1, 2, 4 and 5 are correct.
Only Chief Justices of High Courts are exempted from security checks and not
other Judges of High Courts. So, point 3 is not correct.

So, only four of the above are exempted from security checks at the airports in
India.
Therefore, option (c) is the correct answer.

Question 57. + 2.0 - 0.66

Consider the following statements with reference to the Governor of a State in India:
1. The Draft Constitution of India provided for the direct election of the Governor based
on universal adult suffrage.

2. The Constitution of India requires the President to consult the Chief Minister of the
concerned State while appointing the Governor.

Which of the statements given above is/are correct?

 A. 1 only

B. 2 only

C. Both 1 and 2

 D. Neither 1 nor 2

Question Analytics
4284 USERS 1063 USERS 3221 USERS 31.4 SECS

ATTEMPTED  SOLVED CORRECTLY  SOLVED INCORRECTLY  YOUR TIME

44.98 SECS 0.0 SECS

 AVG. SOLVING TIME  FASTEST SOLVING TIME

Explanation :

The Draft Constitution of India provided for the direct election of the
Governor based on universal adult suffrage. But the Constituent Assembly
opted for the present system of appointment of Governor by the President because
the direct election of the Governor is incompatible with the Parliamentary System
established in the States. So, statement 1 is correct.
Two conventions (not mentioned in the Constitution of India) have also
developed with regards to the appointment of the Governor of a State. First, s/he
should be an outsider, that is, s/he should not belong to the State where s/he is
appointed, so that s/he is free from the local politics. Second, while appointing the
Governor, the President is required to consult the Chief Minister of the State
concerned, so that the smooth functioning of the constitutional machinery in the
State is ensured. So, statement 2 is not correct.

Therefore, option (a) is the correct answer.


Relevance: Recently the Chief Justice of India made a stinging rebuke against the
governors of Punjab and Tamil Nadu in response to serious allegations from their
respective state governments.

Question 58. + 2.0 - 0.66


Two parties from different States which have the same party symbol are contesting for
a State Legislative Assembly seat. Which one of the two will be granted the party
symbol in such a situation?

A. The party with the longest history of functioning and formation

 B. The party with higher vote percentage in their last State Legislative
Assembly elections

C. The party with a higher number of Members of Parliament

Neither of the parties will be able to use their party symbols and will be
 D.
allotted 'free symbol'
Question Analytics
3911 USERS 1999 USERS 1912 USERS 73.2 SECS

ATTEMPTED  SOLVED CORRECTLY  SOLVED INCORRECTLY  YOUR TIME

58.2 SECS 0.0 SECS

 AVG. SOLVING TIME  FASTEST SOLVING TIME

Explanation :

In India, in the case where two parties from different States have the same
election symbol, the parties can contest on their election symbols as long as they
are not fielded against each other in a State Legislative Assembly seat. If they are
fighting against each other, then they will not be able to use their party symbols
and will be allotted 'free symbols' in those constituencies.
The candidates of these parties in such constituencies shall be allotted 'free
symbol' from the list of free symbols as per provisions of Symbols
(Reservation and Allotment) Order, 1968.
While National Parties are free to use their 'reserved symbol' across India, the
recognised State Parties can use their symbols in their States. To use the symbol
outside their State, they have to seek permission from the Election Commission of
India.

Therefore, option (d) is the correct answer.


Relevance: A political party filed a writ petition challenging the Election Commission of
India’s decision to allot the symbols in a State recently.

Question 59. + 2.0 - 0.66

The Speaker of the State Legislative Assembly acts as a quasi-judicial authority in which
one of the following situations?

A. Certifying if a Bill is Money Bill or not

B. Administering oath of office to newly elected members

C. Facilitating discussions on Legislative Bills proposed by the ruling party

Resolving disputes related to disqualification of members under the Tenth


  D.
Schedule of the Constitution of India
Question Analytics
4321 USERS 3830 USERS 491 USERS 24.0 SECS

ATTEMPTED  SOLVED CORRECTLY  SOLVED INCORRECTLY  YOUR TIME

36.17 SECS 0.0 SECS

 AVG. SOLVING TIME  FASTEST SOLVING TIME

Explanation :

Based on the Constitutional provisions, Rules of Business of the House, and the
Conventions, the Speaker’s powers and functions can be divided into four broad
categories which can be to (a) run the business of the House (b) administrative
action (c) quasi-judicial and (d) other functions.
In the quasi-judicial role, the Speaker decides, under the Tenth Schedule of the
Constitution, on the issue of defection of the members from the political party
which influences the composition of the House and the formation/continuation of
the government. While deciding the defection matters, the principles of natural
justice will guide the exercise of power on the procedural aspect. On the
substantive aspect, the Speaker should be guided by the absence of arbitrariness
and the inherent characteristics of impartiality.

Therefore, option (d) is the correct answer.

Knowledge Box

Powers and Functions of Speaker:

The Speaker facilitates the business of the House, ensures equitable


participation of every stakeholder during the discussion, decides on the motions
moved by the members, assists the members to hold the executive accountable,
plays the role of a disciplinarian by suspending/terminating the member or ask
them to withdraw from the House, adjourns the House, expunges the
unparliamentary statements and decides on the nature of the Bill.
On the administrative side, the Speaker heads the Lok Sabha Secretariat,
exercises power over a number of Parliamentary Committees such as the Rules
Committee, the Business Advisory Committee and the General Purposes
Committee and nominates the chairman of various committees in place.
In addition to this, few other powers are vested in the Speaker which includes
the power to exercise a casting vote, to resolve a deadlock over a particular
matter. That is, when the House initiates a voting procedure, he does not cast a
vote in the first instance but shall have and exercise a casting vote in the case of
an equality of votes.

Question 60. + 2.0 - 0.66

In India, Aadhar is mandatory for which of the following purposes?


1. Payment of wages under MGNREGA scheme
2. To avail benefits under Integrated Child Protection and Maternity Benefits schemes

3. Opening of bank accounts by Non-Resident Indians


4. Filing of Income Tax Return

Select the correct answer using the code given below:

A. 1 and 4 only

B. 2 and 3 only

  C. 1, 2 and 4 only

D. 2, 3 and 4 only

Question Analytics
4242 USERS 2567 USERS 1675 USERS 19.7 SECS

ATTEMPTED  SOLVED CORRECTLY  SOLVED INCORRECTLY  YOUR TIME

48.72 SECS 0.0 SECS

 AVG. SOLVING TIME  FASTEST SOLVING TIME

Explanation :

Aadhaar is a 12-digit unique identification number issued by the Unique


Identification Authority of India (UIDAI) to every individual resident of India. It
serves as proof of identity, which is linked to the individual's biometric and
demographic information.
The Central Government has mandated the payment of all MGNREGA (Mahatma
Gandhi National Rural Employment Guarantee Act 2005) wages through an
Aadhaar-Based Payment System (ABPS). All workers are required to furnish
their Aadhaar details. So, point 1 is correct.
All Integrated Child Protection and Maternity Benefits schemes mandatorily
require the Aadhaar card. Women who wish to take advantage of loans, projects
and training opportunities provided by the government now need an Aadhaar. So,
point 2 is correct.
Aadhaar is not a proof of Indian citizenship and does not grant any foreigner the
right to stay in the country in case they are non-compliant with other
requirements. NRIs are not compelled by any mandate to link their bank
accounts with Aadhaar if they do not wish to benefit under section 7 of the
Aadhaar Act. So, point 3 is not correct.
It is mandatory for filing Income Tax Returns (ITR) and allotment of Permanent
Account Number (PAN). Section 139AA of the Income-tax Act, 1961 as introduced
by the Finance Act, 2017 provides for mandatory quoting of Aadhaar/Enrolment
ID of Aadhaar application form, for filing of return of income. So, point 4 is
correct.

Therefore, option (c) is the correct answer.


Relevance: The Central Government has mandated the payment of all wages under the
Mahatma Gandhi National Rural Employment Guarantee Act (MGNREGA) scheme
through an Aadhaar-Based Payment System (ABPS).

Question 61. + 2.0 - 0.66

In the context of the United Nations (UN), consider the following statements:
1. The UN Security Council (UNSC) resolutions are legally binding unlike the UN General
Assembly’s.
2. A UNSC resolution must have the affirmative vote of its nine members including the
five permanent members.

3. A UNSC resolution fails if any one of its five permanent members abstains from
voting.

Which of the statements given above are correct?

 A. 1 and 2 only

B. 2 and 3 only

 C. 1 and 3 only

D. 1, 2 and 3

Question Analytics
3868 USERS 1304 USERS 2564 USERS 79.0 SECS

ATTEMPTED  SOLVED CORRECTLY  SOLVED INCORRECTLY  YOUR TIME

53.9 SECS 0.0 SECS

 AVG. SOLVING TIME  FASTEST SOLVING TIME

Explanation :

All member countries of the United Nations (UN) are part of its General Assembly.
UN Security Council (UNSC) consists of the United States, the United Kingdom,
France, Russia and China. These are the ‘permanent five’ or P5 countries, called so
because the UNSC also has 10 non permanent members who are elected for two
years each based on UN General Assembly (UNGA) elections. This body can pass
resolutions on relevant matters. Only a simple majority (of more than half the
members) is required to pass its resolutions. Also, UNSC resolutions are
legally binding, unlike the UNGA’s. So, statement 1 is correct.
In general, to be adopted, a draft resolution on a non-procedural matter must have
the affirmative vote of nine members of the Council, including the concurring
votes of the five permanent members: China, France, Russian Federation, United
Kingdom of Great Britain and Northern Ireland and the United States of America.
So, statement 2 is correct.
A draft does not pass:
If the it fails to win nine votes, or
If a permanent member casts a negative vote (veto)
Each of the P5 members has the power to veto a vote. It was agreed by the drafters
of the UN Charter (which lays down its governing principles) that if any one of the
five permanent members cast a negative vote in the 15-member UNSC, the
resolution would fail. A member can choose to abstain though, allowing the
resolution to be adopted if it obtains the minimum nine votes. So, statement 3
is not correct.

Therefore, option (a) is the correct answer.


Relevance: Recently, the United Nations General Assembly (UNGA) adopted a resolution
for an immediate ceasefire in the Israel-Hamas conflict in Gaza.

Question 62. + 2.0 - 0.66

Consider the following pairs:

Sl. No. Area of conflict mentioned in news Country where it is located


1. Galmudug : Afghanistan

2. Abyei : South Sudan

3. Amhara : Ethiopia

How many of the above pairs are correctly matched?

A. Only one pair

 B. Only two pairs

C. All three pairs

D. None

Question Analytics
1590 USERS 952 USERS 638 USERS 8.9 SECS

ATTEMPTED  SOLVED CORRECTLY  SOLVED INCORRECTLY  YOUR TIME

20.85 SECS 0.0 SECS

 AVG. SOLVING TIME  FASTEST SOLVING TIME

Explanation :

Galmudug: It is located in central Somalia and has faced inter-clan conflicts


over resources, land and elections. Recently, the Somali National Army and local
soldiers have been fighting to liberate the country from militant groups like al-
Shabaab. So, pair 1 is not correctly matched.
Abyei: It is a region located between Sudan and South Sudan. Both Sudan and
South Sudan claim ownership of Abyei, whose status was unresolved after South
Sudan became independent from Sudan in 2011. Currently, the region is under
the control of South Sudan. Inter-communal and cross-border clashes have
escalated since South Sudan deployed its troops to Abyei in March 2023. So, pair 2
is correctly matched.
Amhara: It is a region in Ethiopia. Fighting erupted here recently between
federal forces and a local militia called Fano, which has accused the
government of undermining the region’s security. The government declared a
state of emergency to respond to the insurgency in the region. So, pair 3 is
correctly matched.

So, only two of the above pairs are correctly matched.


Therefore, option (b) is the correct answer.
Relevance: Recently, Galmudug, Abyei and Amhara have been frequently seen in the
news due to the emergence of local conflicts.

Question 63. + 2.0 - 0.66

Intergovernmental Negotiations (IGN) Framework is often seen in the news in the


context of which one of the following organisations?

  A. United Nations (UN)

B. World Bank (WB)

C. World Economic Forum (WEF)

D. International Monetary Fund (IMF)

Question Analytics
3336 USERS 1685 USERS 1651 USERS 33.0 SECS

ATTEMPTED  SOLVED CORRECTLY  SOLVED INCORRECTLY  YOUR TIME

33.45 SECS 0.0 SECS

 AVG. SOLVING TIME  FASTEST SOLVING TIME

Explanation :

Intergovernmental Negotiations (IGN) Framework is a group of nation-states


working within the United Nations to further reform the United Nations Security
Council (UNSC).
Composition: The IGN is composed of several different international
organisations, namely:
African Union;
G4 nations (India, along with Brazil, Japan and Germany are pressing for a
permanent seat in the reformed UNSC);
Uniting for Consensus Group (UfC), also known as the "Coffee Club";
L.69 Group of Developing Countries;
Arab League; and
Caribbean Community (CARICOM).
The group's conversations are considered "informal" in nature due to the lack of
a single text and thus, United Nations General Assembly rules of procedure don't
apply.

Therefore, option (a) is the correct answer.


Relevance: Recently, France and India called for the early commencement of text-based
negotiations at the Intergovernmental Negotiations (IGN) in the UN.

Question 64. + 2.0 - 0.66

Consider the following statements with reference to Taiwan:

1. It is a representative democracy with a semi-presidential system.

2. It is the largest manufacturer of semiconductors across the World.


3. It is a part of the Chip4 Alliance alongside Japan, South Korea and the United States.
4. India was one of the first countries to establish diplomatic relations with it after
World War II.

How many of the above statements are correct?

A. Only one

B. Only two

  C. Only three

D. All four

Question Analytics
3122 USERS 929 USERS 2193 USERS 31.9 SECS

ATTEMPTED  SOLVED CORRECTLY  SOLVED INCORRECTLY  YOUR TIME

45.26 SECS 0.0 SECS

 AVG. SOLVING TIME  FASTEST SOLVING TIME

Explanation :

Taiwan:

It was established by a Chinese leader named Chiang Kai-shek. After getting


defeated by the Chinese Communist Party, he fled to Taiwan in 1949. It is
generally considered to be part of the People’s Republic of China.

Taiwan has been considered as a semi-presidential country. According to its


constitutional amendments, its President is directly elected, and the Premier
(Prime Minister) and Cabinet are responsible to the Legislature. It has a dual
executive system. So, statement 1 is correct.
It manufactures close to sixty percent of the world's semiconductors and over
90 percent of the most advanced chips that are required for almost all electronic
equipment. So, statement 2 is correct.
The “Chip 4” or “Fab 4” alliance includes four of the world’s top producers of
semiconductors namely the United States of America, Japan, Taiwan and
South Korea. It represents more than 70 per cent of the value of the global
semiconductor industry. It is aimed at enhancing the "security" and "resilience" of
semiconductor supply chains, including by reducing the world's reliance on chips
made in China. So, statement 3 is correct.
It is not a member of the United Nations. India does not have formal diplomatic
ties with Taiwan but both sides have trade and people-to-people relations. India
maintains a One-China policy. So, statement 4 is not correct.

So, only three of the above statements are correct.

Therefore, option (c) is the correct answer.


Relevance: Recently, India and Taiwan reportedly signed an agreement on Labour
cooperation.

Question 65. + 2.0 - 0.66

In which one of the following contexts the terms “Baa2 and Baa3” are mentioned?

A. Quantum computing

  B. Credit ratings

C. Cyber security

D. Gene editing

Question Analytics
3863 USERS 2615 USERS 1248 USERS 6.6 SECS

ATTEMPTED  SOLVED CORRECTLY  SOLVED INCORRECTLY  YOUR TIME

21.08 SECS 0.0 SECS

 AVG. SOLVING TIME  FASTEST SOLVING TIME

Explanation :

A credit rating is an independent assessment of the creditworthiness of a


business or government entity in general terms or with respect to a specific
financial obligation, such as a new bond issue. Baa2 and Baa3 are credit ratings
provided by credit rating agencies.
It assesses how likely an issuer (the borrower) is to pay back investors (the
lenders) and the interest rate it may have to pay in return. They are conferred as
letter grades, ranging from A at the top to C or D at the bottom.
The three major credit rating agencies are Fitch Ratings, Moody's Investors
Service and S&P Global Ratings.
Obligations rated Baa2 are subject to moderate credit risk. They are
considered medium grade and as such may possess certain speculative
characteristics.
In Moody's rating system, Baa3 is a relatively lower investment-grade rating.
It signifies a moderate credit risk for the issuer of the rated financial
instrument or debt.

Therefore, option (b) is the correct answer.


Relevance: Moody's Investors Service has affirmed Reliance Industries Ltd's (RIL's)
Baa2 domestic long-term issuer rating and foreign currency senior unsecured rating.

Question 66. + 2.0 - 0.66

Which one of the following is not a component of the high-powered money of the
Reserve Bank of India (RBI)?

A. Cash in hand with the banks

B. Cash Reserve Ratio (CRR) deposits of banks

 C. Accounts of the International Monetary Fund

 D. Time deposits with the banking system

Question Analytics
4078 USERS 1010 USERS 3068 USERS 29.3 SECS

ATTEMPTED  SOLVED CORRECTLY  SOLVED INCORRECTLY  YOUR TIME

44.72 SECS 0.0 SECS

 AVG. SOLVING TIME  FASTEST SOLVING TIME

Explanation :

Reserve money (M0) is also known as central bank money, base money or high-
powered money. The bulk of the deposit liabilities, along with currency issued by
the central bank, are 'monetary' in that they provide the base for credit creation as
components of reserve money. It plays a crucial role in the determination of
monetary aggregates. Its components are:
Currency in Circulation: It comprises currency with the public and cash
in hand with the banks. The public’s demand for currency is determined by
several factors such as real income, price level, the opportunity cost of
holding currency (i.e., the interest rate on interest-bearing assets) and the
availability of alternative instruments of transactions. e.g., credit/debit cards,
ATMs, cheque payments.
Bankers' Deposits with the Reserve Bank of India (RBI): A major
component of the reserve money is the bankers’ deposits with RBI,
essentially arising out of the Cash Reserve Ratio (CRR) prescribed for the
banks.
"Other" Deposits with RBI: These include deposits of quasi-government and
other financial institutions including primary dealers, balances in the
accounts of foreign Central Banks and Governments, accounts of
international agencies such as the International Monetary Fund, etc.
M1 is a monetary aggregate used for assessing the money supply in the economy.
It is referred to as narrow money.
M1 = Currency with the Public + Demand Deposits with the Banking
System + ‘Other’ Deposits with RBI
Thus, M1 = Currency with the Public + Current Deposits with the Banking
System + Demand Liabilities Portion of Savings Deposits with the Banking
System + ‘Other’ Deposits with RBI
M2 = M1 + Time Liabilities Portion of Savings Deposits with the Banking
System + Certificates of Deposit issued by Banks + Term Deposits of residents
with a contractual maturity of up to and including one year with the Banking
System

Thus, M2 = Currency with the Public + Current Deposits with the Banking System +
Savings Deposits with the Banking System + Certificates of Deposits issued by Banks +
Term Deposits of residents with a contractual maturity up to and including one year
with the Banking System + ‘Other’ Deposits with RBI.

M3 = M2 + Term Deposits of residents with a contractual maturity of over one


year with the Banking System + Call/Term borrowings from ‘Non-depository’
financial corporations by the Banking System.
‘Other’ deposits with RBI comprise mainly:
deposits of quasi-government and other financial institutions including
primary dealers
balances in the accounts of foreign Central banks and Governments
accounts of international agencies such as the International Monetary Fund,
etc.

Therefore, option (d) is the answer.


Knowledge Box

Government balances with the Reserve Bank of India (RBI) are considered
non-monetary because the State is treated as a part of the money-issuing sector
along with the central bank because of its ability to create money by fiat.
M0 is computed every week while M1 is calculated on a fortnightly basis.

Question 67. + 2.0 - 0.66

Which of the following securities qualify for the purpose of Statutory Liquidity Ratio
(SLR) maintained by banks in India?
1. State Development Loans

2. Bonds issued to the Food Corporation of India


3. Treasury Bills

4. Bank Recapitalisation Bonds


5. Cash Management Bills
Select the correct answer using the code given below:

A. 1, 2 and 4 only

B. 3 and 5 only

 C. 1, 3 and 5 only

D. 1, 2, 3, 4 and 5

Question Analytics
3376 USERS 1179 USERS 2197 USERS 45.4 SECS

ATTEMPTED  SOLVED CORRECTLY  SOLVED INCORRECTLY  YOUR TIME

60.15 SECS 0.0 SECS

 AVG. SOLVING TIME  FASTEST SOLVING TIME

Explanation :

Statutory Liquidity Ratio (SLR) is a minimum percentage of deposits that a


commercial bank has to maintain in the form of liquid cash, gold or other
securities. It is basically the reserve requirement that banks are expected to keep
before offering credit to customers. These are not reserved with the Reserve Bank
of India (RBI), but with banks themselves. The SLR was prescribed by the
Banking Regulation Act, 1949.
State Development Loans (SDLs) are market borrowings by State
Governments. Each State is allowed to issue securities up to a certain limit each
year. SDLs qualify for SLR status. So, point 1 is correct.
Under the market borrowing program, the Government of India issues, from time
to time, special bonds/securities to entities like Oil Marketing Companies, Fertilizer
Companies, the Food Corporation of India, etc. (popularly called oil bonds,
fertiliser bonds and food bonds respectively) as compensation to these companies
in lieu of cash subsidies. These securities are not eligible as SLR securities but
are eligible as collateral for market repo transactions. So, point 2 is not correct.
Treasury Bills or T-Bills are the instruments through which short term
borrowings are solicited by the Government of India. They are promissory notes
issued at a discount to their face value for a fixed time period e.g., 91 days, 182
days and 364 days. T-Bills qualify for SLR securities. So, point 3 is correct.
The Government of India had issued Bank Recapitalisation Bonds to specific
Public Sector Banks in 2018. These securities are non-transferable and are not
eligible investment in pursuance of any statutory provisions or directions
applicable to investment banks, including the SLR. So, point 4 is not correct.
Cash Management Bills (CMB) have been introduced by the Reserve Bank of
India to meet temporary cash flow mismatches of the Government of India.
CMBs are non-standard, discounted instruments issued for maturities less than 91
days. Investment by banks in CMBs is treated as an eligible investment in
Government Securities for SLR purpose. So, point 5 is correct.

Therefore, option (c) is the correct answer.

Relevance: According to the RBI, banks have increased their investments in Statutory
Liquidity Ratio (SLR) securities by 4.7 percent in the first quarter of FY24.

Question 68. + 2.0 - 0.66

In India, which one of the following authorities is responsible for the compilation of
data for Balance of Payments (BoP)?

A. Department of Economic Affairs

  B. Reserve Bank of India

C. Ministry of Commerce and Industry

D. National Statistical Office

Question Analytics
4230 USERS 2112 USERS 2118 USERS 21.3 SECS
ATTEMPTED  SOLVED CORRECTLY  SOLVED INCORRECTLY  YOUR TIME

32.83 SECS 0.0 SECS

 AVG. SOLVING TIME  FASTEST SOLVING TIME

Explanation :

Balance of Payments (BoP) is a statistical statement summarising all the external


transactions (receipts and payments) on current and capital accounts in which a
country is involved over some time. As the BoP shows the total assets and
obligations over a time period, it always balances.
The compilation and dissemination of BoP data is the prime responsibility of
the Reserve Bank of India (RBI). In India, the compilation of BoP statistics is
broadly consistent with the guidelines contained in the BoP Manual, 5th Edition, of
the International Monetary Fund (BPM5). The format of the presentation of the
BoP data for the period from 1990-91 is based on the recommendations of the
High-Level Committee under the chairmanship of Dr. C. Rangarajan on Balance of
Payments, 1993.

Therefore, option (b) is the correct answer.

Relevance: According to recent data released by the Reserve Bank of India (RBI), the
Current Account Deficit (CAD) was 3.8 per cent of Gross Domestic Product (GDP).

Question 69. + 2.0 - 0.66

Consider the following situations:


1. Increase in the Foreign Portfolio Investment (FPI) limit

2. Decrease in Dollar-Rupee swap

3. Increase in the External Commercial Borrowings (ECBs)


4. Restriction on the Foreign Capital Inflows

How many of the above are most likely to result in the appreciation of the Rupee?

 A. Only one

 B. Only two

C. Only three

D. All four

Question Analytics
3848 USERS 2266 USERS 1582 USERS 51.7 SECS
ATTEMPTED  SOLVED CORRECTLY  SOLVED INCORRECTLY  YOUR TIME

72.96 SECS 0.0 SECS

 AVG. SOLVING TIME  FASTEST SOLVING TIME

Explanation :

Fall in the exchange rate i.e increase in the external value of domestic currency, due to
additional demand for home currency (or less supply of home currency) or less demand
for (or more supply of) foreign currency is called appreciation.

Increasing the Foreign Portfolio Investment (FPI) limit generally attracts more
foreign capital inflows, leading to increased demand for INR, potentially causing
appreciation. When the FPI limit is increased, it indicates greater confidence of
foreign investors in the Indian economy, leading to higher inflows of foreign
funds. This can contribute to an increase in the demand for Rupee, thereby
causing its appreciation. So, point 1 is correct.
Decrease in Dollar-Rupee swap: It’s a forex tool whereby the central bank uses
its currency to buy another currency or vice versa. In a Dollar–Rupee buy/sell
swap, the central bank buys dollars (US dollars or USD) from banks in exchange
for Indian Rupees (INR) and immediately gets into an opposite deal with banks
promising to sell dollars at a later date. In a dollar–rupee sell/buy swap it sells USD
in exchange for INR and promises to buy dollars from banks after some years.
When there is decrease in the dollar-rupee swap, it implies less demand for
rupee globally which results in the depreciation of the domestic currency.
Thus, the rupee will depreciate. So, point 2 is not correct.
External Commercial Borrowing (ECB) is a type of loan in foreign currencies,
made by non-resident lenders. Thus, easing conditions of the ECB helps in
receiving more loans in foreign currencies. This would increase the inflow of
forex, leading to rupee appreciation. So, point 3 is correct.
Capital inflows result in a buildup of foreign exchange reserves. As these
reserves are used to buy domestic currency, the domestic monetary base expands
without a corresponding increase in production: too much money begins to chase
too few goods and services. This creates appreciation of the domestic currency.
Restricting the foreign capital inflows will inverse the situation and thus lead to
depreciation of the rupee. So, point 4 is not correct.

So, only two of the above are most likely to result in the appreciation of the Rupee.

Therefore, option (b) is the correct answer.

Question 70. + 2.0 - 0.66


The Catastrophe Containment and Relief Trust (CCRT) is the:

World Health Organization’s (WHO) provision for medical aid during public
A.
health emergency

International Monetary Fund’s (IMF) debt relief to the countries affected by


 B.
natural disasters

Regional Anti-Terrorist Structure’s (RATS) framework to contain terrorism in


C.
member countries

United Nations Peacekeeping Forces’ relief services to communities affected


 D.
in conflict zones

Question Analytics
3209 USERS 1753 USERS 1456 USERS 26.8 SECS

ATTEMPTED  SOLVED CORRECTLY  SOLVED INCORRECTLY  YOUR TIME

34.89 SECS 0.0 SECS

 AVG. SOLVING TIME  FASTEST SOLVING TIME

Explanation :

The Catastrophe Containment and Relief Trust (CCRT) was established by the
International Monetary Fund (IMF) to provide grants for debt relief to the
poorest and most vulnerable countries affected by catastrophic natural disasters
or public health crises, enabling them to meet their debt service obligations to the
IMF. This relief frees up resources of these countries, helping them meet
exceptional balance of payments needs created by the disaster and to pay for
containment and recovery.
CCRT assistance is available to countries if they are eligible to borrow from the
IMF’s Poverty Reduction and Growth Trust (PRGT) and their per-capita income
is less than the International Development Association (IDA) operational cutoff.
Countries qualify for relief if a natural disaster has directly affected at least
one-third of the population, is estimated to have destroyed more than a quarter
of the country’s productive capacity or has caused damage deemed to exceed 100
percent of GDP.
Eligibility for public health disaster relief covers life-threatening epidemics
and pandemics that has affected several areas of their country, has the
potential to spread or is already spreading to other countries, and is causing
significant economic disruption.

Therefore, option (b) is the correct answer.


Relevance: Managing director of the International Monetary Fund (IMF) has identified
domestic resource mobilisation to accelerate decarbonisation.

Question 71. + 2.0 - 0.66

Which of the following can be used as trademarks under the Trade Marks Act, 1999?

1. Letters or numerals
2. Sound marks

3. Combination of colours

4. Name of chemical element


5. Shape of goods

Select the correct answer using the code given below:

A. 1, 2 and 4 only

B. 3, 4 and 5 only

 C. 1, 2, 3 and 5 only

 D. 1, 2, 3, 4 and 5

Question Analytics
3253 USERS 1458 USERS 1795 USERS 12.8 SECS

ATTEMPTED  SOLVED CORRECTLY  SOLVED INCORRECTLY  YOUR TIME

45.29 SECS 0.0 SECS

 AVG. SOLVING TIME  FASTEST SOLVING TIME

Explanation :

The Trade Marks Act, 1999 adheres to Trade-Related Aspects of Intellectual


Property Rights (TRIPS), 1994.
Types of trademarks that may be registered in India:
Any name (including personal or surname of the applicant or predecessor in
business or the signature of the person), which is not unusual for trade to
adopt as a mark.
An invented word or any arbitrary dictionary word or words, not being
directly descriptive of the character or quality of the goods/service.
Letters or numerals or any combination thereof. So, point 1 is correct.
The right to proprietorship of a trademark may be acquired by either
registration under the Act or by use in relation to particular goods or service.
Devices, including fancy devices or symbols
Monograms
Combination of colours or even a single color in combination with a word
or device. So, point 3 is correct.
Shape of goods or their packaging. So, point 5 is correct.
Marks constituting a 3- dimensional sign.
Sound marks when represented in conventional notation or described in
words by being graphically represented. So, point 2 is correct.
The Trade Marks Act, 1999 prohibits the registration of trade marks that include
a frequently used and accepted name of any single chemical element or
chemical compound in relation to a chemical product or preparation. A word
that has been declared as an International Non-Proprietary Name (INN) by WHO
(World Health Organization) or that is deceptively similar to such a name as
informed by the Registrar is not permitted to be registered. So, point 4 is not
correct.

Therefore, option (c) is the correct answer.

Knowledge Box

Signature Trademarks registered in India:

While the registration of a trademark is not required by law, it is recommended


since unregistered trademarks receive little protection. Celebrities often register
their signatures, names, initials etc.

Question 72. + 2.0 - 0.66

Consider the following fish species:

1. Guppy
2. Aplocheilus panchax

3. Oryzias melastigma

4. Gambusia affinis

How many of the above can be used to control the mosquito spread?

 A. Only one

B. Only two

C. Only three

 D. All four

Question Analytics
1998 USERS 955 USERS 1043 USERS 11.7 SECS

ATTEMPTED  SOLVED CORRECTLY  SOLVED INCORRECTLY  YOUR TIME

17.86 SECS 0.0 SECS

 AVG. SOLVING TIME  FASTEST SOLVING TIME

Explanation :

Several species of both indigenous and exotic fish are used to control the mosquito
population because of their larvivorous potential.

Poecilia reticulata: It is also called ‘Guppy’ and is an exotic fish introduced in


India. It cannot survive in cold water below 10°C. It is a surface feeder and a
single fish eats about 80 to 100 mosquito larvae in 24 hours. So, point 1 is correct.
Aplocheilus panchax: It is an indigenous variety and widely distributed in
Bengal, Bihar, Orissa, Assam, Punjab, Uttar Pradesh, Madhya Pradesh and
Rajasthan. It is a potential larvivorous fish in controlling several vector species
in different types of natural and man-made habitats. It is used for filariasis
vector control and is effective on both anophelines and culicines. So, point 2 is
correct.
Oryzias melastigma: It is an indigenous fish and is widely distributed in Bengal,
Odisha and Tamil Nadu. It is a surface feeder found in both still and running
waters. It is suitable for open shallow water stretches especially in rice fields for
control of mosquitoes causing Japanese B encephalitis. So, point 3 is correct.
Gambusia affinis: It is an exotic species and a surface feeder, hence it is
suitable for feeding on both anophelines and culicines. The fish has been used
extensively in different parts of the country for the control of Anopheles
stephensi breeding particularly in urban areas. So, point 4 is correct.
Some other species of larvivorous fish include Aphanius dispar, Colisa fasciatus,
Chanda nama, Carassius auratus, Xenentodon cancila etc.

So, all four of the above can be used to control the mosquito spread.

Therefore, option (d) is the correct answer.


Relevance: The Andhra Pradesh Government has released approximately 10 million
Gambusia fish into the state’s water bodies to combat mosquito-borne diseases like
malaria and dengue.

Question 73. + 2.0 - 0.66


Consider the following statements with reference to the nomenclature of the tropical
cyclones in the North Indian Ocean:

1. The World Meteorological Organisation (WMO) is responsible for naming all tropical
cyclones.

2. The names of tropical cyclones once used, cannot be used again.


3. The name of the cyclone should not exceed eight letters.

How many of the above statements are correct?

A. Only one

  B. Only two

C. All three

D. None

Question Analytics
3885 USERS 1938 USERS 1947 USERS 27.9 SECS

ATTEMPTED  SOLVED CORRECTLY  SOLVED INCORRECTLY  YOUR TIME

43.62 SECS 0.0 SECS

 AVG. SOLVING TIME  FASTEST SOLVING TIME

Explanation :

The Regional Specialized Meteorological Centre (RSMC) New Delhi Tropical


Cyclone Center is responsible for naming the tropical cyclones that have formed
over the Bay of Bengal and the Arabian Sea when they have reached the relevant
intensity. So, statement 1 is not correct.
Criteria for selection of tropical cyclones names by the Panel on Tropical Cyclones
(PTC) member countries:
The proposed name should be neutral to (a) politics and political figures (b)
religious believes, (c) cultures and (d) gender
Name should be chosen in such a way that it does not hurt the sentiments of
any group of population over the globe
It should not be very rude and cruel in nature
It should be short, easy to pronounce and should not be offensive to any
member
The maximum length of the name will be eight letters. So, statement 3 is
correct.
The proposed name should be provided along with its pronunciation,
meaning and voice over
The Panel reserves the right to reject any name, if any of the criteria above is
not satisfied.
The finalised names may also be reviewed during the course of time of
implementation with the approval of PTC in its annual session, in case any
reasonable objection is raised by any member
The names of tropical cyclones over the North Indian Ocean will not be
repeated, once used it will cease to be used again. The name should be
new. It should not be there in the already existing list of any of the RSMCs
worldwide including RSMC New Delhi. So, statement 2 is correct.

So, only two of the above statements are correct.

Therefore, option (b) is the correct answer.

Relevance: Cyclone Michaung made landfall over Nellore in Andhra Pradesh as a


super-cyclonic storm.

Question 74. + 2.0 - 0.66

‘Bromeliads’, sometimes seen in the news, are:

A. carnivorous plants that capture insects for nutrition

 B. plants that can store water in their leaves

C. frost-resistant plants that can thrive in cold climates

 D. plants that have adapted to grow in saline conditions

Question Analytics
2684 USERS 976 USERS 1708 USERS 21.9 SECS

ATTEMPTED  SOLVED CORRECTLY  SOLVED INCORRECTLY  YOUR TIME

30.91 SECS 0.0 SECS

 AVG. SOLVING TIME  FASTEST SOLVING TIME

Explanation :

Bromeliads are special plants that store water in their leaves. They are native
to tropical North and South America. The commercial pineapple (Ananas
comosus) is native to southern Brazil and Paraguay.
They typically have bright red, orange, purple, or blue flowers, and can grow in
several different ways: they can be terrestrial, saxicolous (growing on rocks) or
epiphytic (growing on other plants and trees).
Many bromeliads are tropical epiphytes. They have a special form of
photosynthesis that uses a variation of the more usual biochemical pathways to
allow them to use water more efficiently.
They have stiff, overlapping leaves which hold rainfall like buckets. Leaves and
debris fall into these reservoirs and help algae and other single-celled organisms
to grow, which in turn feed mosquitoes, insect larvae, and other organisms. The
bromeliad is like a small ecosystem in itself. Animals such as tree frogs, snails,
flatworms, tiny crabs, and salamanders might spend their entire lives inside them.
Animals like frogs use these pockets of water to lay their eggs.

Therefore, option (b) is the correct answer.

Relevance: As per a recent study, Bromeliads are at a greater threat due to rapid
environmental change.

Question 75. + 2.0 - 0.66

Consider the following species:


1. Shama

2. Dhyal

3. Pied wagtail
4. Indian robin

How many of the above are songbirds found in India?

A. Only one

B. Only two

C. Only three

 D. All four

Question Analytics
1631 USERS 752 USERS 879 USERS 36.7 SECS

ATTEMPTED  SOLVED CORRECTLY  SOLVED INCORRECTLY  YOUR TIME

16.17 SECS 0.0 SECS

 AVG. SOLVING TIME  FASTEST SOLVING TIME

Explanation :

India is home to some songbirds capable of competing with the best. There are
numerous songbirds in India.
The shama (Cittocincla macrura) is one of the well known songbirds. It is not
likely to be heard in any large town. It lives among shady hills and the great
forests of India. So, point 1 is correct.
A fine songbird, nearly related to the shama, and found in nearly all parts of India,
is the magpie robin or dhyal (Copsychus saularis). It has the confiding habits of
the robin, the bright colouring of the magpie, and the voice of the canary. It is
nearly always found near human habitations. So, point 2 is correct.
The small pied wagtail is the most attractive of birds that nest around human
habitations and brings joy with its quiet, sweet voice throughout the year. So,
point 3 is correct.
The Indian robin is a singing bird found all over India, but robins south of the
Godavari River differ slightly from those which dwell in Northern India. So, point
4 is correct.

So, all four of the above are songbirds found in India.

Therefore, option (d) is the correct answer.


Relevance: The UN report stated that nearly half of the world's migratory species
including songbirds are on the decline.

Question 76. + 2.0 - 0.66

Which one of the following species is recently seen in the news in context of efforts
taken to preserve the Amazon rainforest?

 A. Stingless bee

B. Hoverfly

 C. Fig Wasp

D. Elephant Hawk Moth

Question Analytics
1967 USERS 736 USERS 1231 USERS 14.1 SECS

ATTEMPTED  SOLVED CORRECTLY  SOLVED INCORRECTLY  YOUR TIME

26.84 SECS 0.0 SECS

 AVG. SOLVING TIME  FASTEST SOLVING TIME

Explanation :

A recent mission has commenced in Peru aimed at preserving the Stingless Bee,
which is indigenous to the tropics. These pollinators are playing a pivotal role in
one of the most recent endeavours to safeguard the Amazon rainforest.
Stingless bees are native to the Amazon, unlike the more familiar but invasive
honey bees from Africa and Europe that have spread through the Americas. The
most obvious difference, perhaps, is that stingless bees don’t sting. Their honey,
which is runny enough to be drunk like a liquid and is said to have a citrusy
aftertaste, is used by many Indigenous Peruvians as a natural medicine.

Therefore, option (a) is the correct answer.


Relevance: Recently, in Peru, a Mission started to Save the Stingless Bee - Native to the
tropics, these pollinators are taking a lead role in one of the latest efforts to conserve
the Amazon rainforest.

Question 77. + 2.0 - 0.66

Consider the following statements with reference to bats:

1. Some bat species feed on insects like moths.


2. Some bat species use echolocation to navigate dark caves.

3. Some bat species hibernate to survive harsh winters.

4. Some bat species can be found in polar regions.


How many of the above statements are correct?

A. Only one

B. Only two

  C. Only three

D. All four

Question Analytics
3896 USERS 1181 USERS 2715 USERS 20.5 SECS

ATTEMPTED  SOLVED CORRECTLY  SOLVED INCORRECTLY  YOUR TIME

25.34 SECS 0.0 SECS

 AVG. SOLVING TIME  FASTEST SOLVING TIME

Explanation :

Bats are the only mammals in the world that can fly. There are two main types
of bats: microbats and megabats. Most bats are microbats, which eat insects
like moths that come out at night. Vampire bats are the only species of microbats
that feed on blood rather than insects. So, statement 1 is correct.
To navigate dark caves and hunt after dark, microbats rely on echolocation, a
system that allows them to locate objects using sound waves. They echolocate
by making a high-pitched sound that travels until it hits an object and bounces
back to them. This echo tells them an object’s size and how far away it is. So,
statement 2 is correct.
In contrast, megabats live in the tropics and eat fruit, nectar, and pollen.
They have larger eyes and a stronger sense of smell than microbats but have
smaller ears because they don’t echolocate.
Bears and bats are the two most well-known hibernators to survive harsh
winters. Some bat species like the spotted bat survive by migrating in search of
food to warmer areas when it gets chilly. So, statement 3 is correct.
Bats can be found on nearly every part of the planet except in extreme deserts
and polar regions. They spend their daylight hours hiding in roosts around the
tropics, dense forests, and wetlands. Roosts are where bats go to rest, usually in
cracks and crevices that keep them hidden and protected. The most common
roosts are existing structures such as caves, tree hollows and old buildings. So,
statement 4 is not correct.
Vampire bat saliva includes an anticoagulant (a substance that stops blood clots
from forming) that might be utilised to treat human heart ailments.

So, only three of the above statements are correct.

Therefore, option (c) is the correct answer.

Relevance: Recently, a research was published related to the immune response of bats.

Question 78. + 2.0 - 0.66

Consider the following species:

1. Dhole (wild dog)

2. Sloth bear
3. Mouse deer

4. Lion-tailed macaque

5. Wild ass
How many of the above are most likely to be observed if you visit the Bandipur
National Park?

A. Only two

B. Only three
 C. Only four

D. All five

Question Analytics
3027 USERS 1073 USERS 1954 USERS 10.4 SECS

ATTEMPTED  SOLVED CORRECTLY  SOLVED INCORRECTLY  YOUR TIME

31.7 SECS 0.0 SECS

 AVG. SOLVING TIME  FASTEST SOLVING TIME

Explanation :

Bandipur National Park is located in Karnataka in the Western Ghats. It is an


important part of the Nilgiri Biosphere Reserve that constitutes Karnataka’s
Rajiv Gandhi National Park (Nagarahole) to its Northwest, Tamil Nadu’s
Mudumalai Wildlife Sanctuary to its South, and Kerala’s Wayanad Wildlife
Sanctuary to its Southwest.
Several endangered species such as Asian Wild Elephants, Tigers, sloth bears,
gaurs, dhole (wild dog) Indian rock pythons, jackals, muggers, and four-
horned antelopes can be spotted in this national park. Bandipur also shelters
sambar, mouse deer, chital and the rare flying lizard. So, points 1, 2 and 3 are
correct.
It is also home to several primate species, including the endangered Nilgiri langur
and the lion-tailed macaque. So, point 4 is correct.
Wild asses are found in the Rann of Kutch of Gujarat. So, point 5 is not
correct.

So, only four of the above are most likely to be observed if you visit the Bandipur
National Park.

Therefore, option (c) is the correct answer.


Relevance: Bandipur National Park, which is home to more than 1100 elephants as per
the last estimation, is set to get a task force to deal with conflict situations.

Question 79. + 2.0 - 0.66

Consider the following statements:

1. All transgenic organisms are Genetically Modified Organisms (GMOs), but not all
GMOs are transgenic.

2. The Ministry of Environment, Forest and Climate Change is the final authority to
approve the testing of transgenic crops in India.
3. The import of GMOs for food and feed is regulated by the Seeds Act of 1966.

How many of the above statements are correct?

  A. Only one

B. Only two

C. All three

D. None

Question Analytics
3191 USERS 979 USERS 2212 USERS 36.9 SECS

ATTEMPTED  SOLVED CORRECTLY  SOLVED INCORRECTLY  YOUR TIME

40.69 SECS 0.0 SECS

 AVG. SOLVING TIME  FASTEST SOLVING TIME

Explanation :

Transgenic refers to an organism or cell whose genome has been altered by the
introduction of one or more foreign DNA sequences from another species by
artificial means. Transgenic organisms are generated in the laboratory for
research purposes.
Genetically Modified Organism (GMO) and transgenic organism are two terms
often used interchangeably. Both types of organisms have an altered genome that
has been modified artificially. However, there is a slight difference between GMO
and transgenic organisms. Although both have altered genomes, a transgenic
organism is a GMO containing a DNA sequence or a gene from a different species.
Thus, all transgenic organisms are GMOs, but not all GMOs are transgenic. So,
statement 1 is correct.
Transgenic animals have been generated to understand normal physiological
processes such as metabolism and blood cell production. They have been used to
model human diseases and to develop new treatments. Transgenic crops have
been produced that are resistant to infectious diseases and grow faster.
Agriculture is a State subject as per the Seventh Schedule of the Constitution of
India. It implies that companies interested in testing transgenic crops need
approvals from the States to conduct such tests. Only Haryana permitted such
tests. So, statement 2 is not correct.
The import of Genetically Modified Organisms for research & development,
food, feed, processing in Bulk and for environmental release is governed by the
provisions of the Environment Protection Act, 1986 and Rules 1989.
Environmental release or field application will be allowed only with the approval
of the Genetic Engineering Approval Committee (GEAC). So, statement 3 is not
correct.

So, only one of the above statements is correct.


Therefore, option (a) is the correct answer.

Knowledge Box

Genetic Engineering Appraisal Committee (GEAC):

It functions under the Union Ministry of Environment, Forest and Climate


Change and is responsible for the appraisal of activities involving large-scale use
of hazardous microorganisms and recombinants in research and industrial
production from the environmental angle.
It is also responsible for the appraisal of proposals relating to the release of
Genetically Engineered (GE) organisms and products into the environment
including experimental field trials.

Relevance: Recently, a few States have deferred a proposal to test a new kind of
transgenic cotton seed that makes it resistant to pink bollworms.

Question 80. + 2.0 - 0.66

Consider the following activities:

1. Injection of carbonic acid into ultramafic rocks


2. Coal gasification

3. Carbonation of concrete

4. Ocean alkalinization
Which of the above are often considered for Carbon Mineralization?

A. 1 and 2 only

 B. 1, 3 and 4 only

C. 3 and 4 only

D. 1, 2, 3 and 4

Question Analytics
2695 USERS 1039 USERS 1656 USERS 13.3 SECS

ATTEMPTED  SOLVED CORRECTLY  SOLVED INCORRECTLY  YOUR TIME

40.08 SECS 0.0 SECS


 AVG. SOLVING TIME  FASTEST SOLVING TIME

Explanation :

‘Carbon Mineralization’ is a natural phenomenon wherein atmospheric carbon


dioxide (CO2) reacts with certain minerals within rocks, efficiently removing CO2
from the atmosphere. This process securely stores the carbon in the form of
carbonates. This process occurs naturally over hundreds or thousands of years,
but can be sped up to provide a permanent solution for Carbon Sequestration in
order to mitigate global warming as follows:
Carbonic acid (H2CO3) injection into Mafic or Ultramafic rocks involves

dissolving concentrated CO2 in water to create carbonated water or


carbonic acid (H2CO3). The resulting solution is injected into wells drilled
into mafic or ultramafic rocks, known for their high reactivity with CO2.
Within these rocks, carbonate minerals form as a result of the reaction,
effectively capturing and sequestering the CO2. Subsequently, the water

depleted of CO2 is brought back to the Earth's surface. So, point 1 is correct.
Carbonation of concrete is the chemical reaction between carbon dioxide
(CO2) in the air and calcium hydroxide and hydrated calcium silicate in the
concrete to give mainly carbonates. Injecting captured CO2 into concrete

before its hardening forms calcium carbonate, enhancing concrete strength


and reducing cement usage. Additionally, this process securely stores CO2,

contributing to long-term Carbon Sequestration. So, point 3 is correct.


Ocean alkalinization is a carbon removal method where alkaline
substances are introduced into seawater to boost its natural carbon
absorption capacity. These substances, like olivine minerals or industrial
byproducts like lime, chemically react with dissolved carbon dioxide (CO2) in

the ocean, converting it into stable bicarbonate and carbonate molecules.


This process enhances the ocean's ability to absorb CO2 from the

atmosphere, aiding in restoring equilibrium. So, point 4 is correct.


Coal Gasification is a process in which coal is partially oxidised by air, oxygen,
steam or carbon dioxide under controlled conditions to produce a fuel gas. The
hot fuel gas is cooled in heat exchangers, with the production of steam, and
cleaned before combustion in a gas turbine. It is not considered for Carbon
Mineralization. So, point 2 is not correct.

Therefore, option (b) is the correct answer.


Relevance: The Union Cabinet has approved India’s updated Nationally Determined
Contribution (NDC) to be communicated to the United Nations Framework Convention
on Climate Change (UNFCCC).

Question 81. + 2.0 - 0.66

In context of history of medieval India, the officers ‘qazi- ul- quzat, tui-begis and Mir-
adl’ were associated with:

 A. intelligence and espionage

B. revenue collection

 C. administration of justice

D. foreign affairs

Question Analytics
3563 USERS 2803 USERS 760 USERS 18.4 SECS

ATTEMPTED  SOLVED CORRECTLY  SOLVED INCORRECTLY  YOUR TIME

25.16 SECS 0.0 SECS

 AVG. SOLVING TIME  FASTEST SOLVING TIME

Explanation :

Qazi-ul-qazat was the chief official with a position next to the emperor. He
decided cases in accordance with the Islamic law and also appointed Qazis in
different parts of empires. The king appointed the chief qazi sadr who possessed
the powers of judge and had the power of appointing subordinate qazis in the
dominations, though the king’s sanction was necessary in all such appointments.
The king also exercised his power to appoint more than one judge in a city and
their duties were accordingly defined. These were the qazis and mir adls. In all
big cities and towns the two existed side by side.
Qazi –i- mamalik or Qazi- ul- quzat was the name given to the Chief Judge of the
sultanate. The secular judge was called Mir- adl during the Mughal era. He
represented the judge on account of the Emperor. It was his sole duty and
responsibility to implement Qazi’s decisions and he was also called to make
unprejudiced and individual inquiries.
The two officers were appointed by Akbar called Tui-begis. It was their duty to
look into obedience to the law and a very affordable amount of money was fixed
as their remuneration or fee.
Therefore, option (c) is the correct answer.

Knowledge Box

Mir Saman or Khan-i- Saman was the minister of the royal Family and looked
after royal buildings, roads, parks, karkhanas etc. He was incharge of the
imperial household including the supply of all the provisions and articles for the
use of inmates of the haram or the female apartments.
Amin, Qanungo were appointed as revenue officials.
Mir Bakshi headed the military department, nobility, information and
intelligence agencies.

Question 82. + 2.0 - 0.66

Consider the following:

1. Marco Polo

2. Al-Biruni
3. Nicolo De Conti

4. Ibn Battuta

Which one of the following is the correct chronological order of the above foreign
travellers' visit to India?

A. 2–3–1–4

B. 1–2–4–3

 C. 2–1–4–3

D. 4–2–3–1

Question Analytics
3264 USERS 1742 USERS 1522 USERS 6.9 SECS

ATTEMPTED  SOLVED CORRECTLY  SOLVED INCORRECTLY  YOUR TIME

43.71 SECS 0.0 SECS

 AVG. SOLVING TIME  FASTEST SOLVING TIME

Explanation :

Al-Biruni: He was born in Khwarizm (Uzbekistan).. He was a scholar who was


well versed in Syriac, Arabic, Persian, Hebrew and Sanskrit. In 1017, Sultan
Mahmud Ghazni invaded Khwarizm and took many scholars including Al-Biruni
to his capital, Ghazni. Owing to the inclusion of Punjab in the Ghaznavid Empire,
Al-Biruni travelled widely in Punjab and other Northern parts of India from 1024-
1030 AD. He translated several Sanskrit works, including Patanjali’s work on
Grammar and also authored ‘Kitab-ul-Hind’.
Marco Polo: He was born in the republic of Venice. He was an Italian merchant,
adventurer and writer who wrote “The Travels of Marco Polo”, an account of his
travels from one country to another. He visited the Kakatiya kingdom in around
1289 AD during the reign of Queen Rudramadevi.
Ibn Battuta: He was a Moroccan traveller who was born in Tangier. Throughout
his life, he travelled extensively in Syria, Iraq, Persia, Yemen, Oman and a few
trading ports on the coast of East Africa. He came to India in 1332–33 AD during
the reign of Muhammad Bin Tughlaq who was impressed by his scholarship, and
appointed him as “Qazi” or judge of Delhi. He wrote a book of travels in Arabic
called ‘Rihla’.
Nicolo De Conti: He was an Italian merchant and explorer. He crossed the
Arabian Sea to Cambay in Gujarat and entered India in 1420-1421 AD. He travelled
to the Vijayanagara kingdom during the reign of Deva Raya I. He referred to the
Telugu language as “Italian of the East”. He described South-East Asia as
“overtaking all other regions in terms of wealth, culture and magnificence”.

So, the correct chronological order of the above mentioned foreign travellers visit
to India is Al-Biruni - Marco Polo - Ibn Battuta - Nicolo De Conti.

Therefore, option (c) is the correct answer.

Question 83. + 2.0 - 0.66

Consider the following statements with reference to Adi Shankaracharya:


1. He proposed “Advaita Vedanta” to promote non-dualism of reality.

2. He set up four cardinal mathas in four corners of India.

3. He authored the books “Gita Bhasya” and “Vedantasara”.


4. He was a contemporary of Ramanujacharya.

How many of the above statements are correct?

A. Only one

  B. Only two

C. Only three

D. All four
Question Analytics
3330 USERS 964 USERS 2366 USERS 21.1 SECS

ATTEMPTED  SOLVED CORRECTLY  SOLVED INCORRECTLY  YOUR TIME

31.74 SECS 0.0 SECS

 AVG. SOLVING TIME  FASTEST SOLVING TIME

Explanation :

Adi Shankaracharya is one of the most important figures in Hinduism and one of
the foremost proponents of Advaita Vedanta. He was born in Kerala in the
eighth century CE. He had set up four cardinal mathas at Badrinath, Sringeri,
Puri and Dwarka in the North, South, East and West of India respectively. So,
statement 2 is correct.
Advaita Vedanta is a school of Hindu philosophy and spiritual discipline. It
articulates an ontological position of radical nondualism. It posits that all that we
perceive is ultimately illusory (maya) and that the principle of Brahman is the
only true reality of all things, transcending empirical plurality. The fundamental
idea of Advaita Vedanta lies in the unity of Atman or individual consciousness
and Brahman or the ultimate reality. So, statement 1 is correct.
He is also identified as the author of multiple works, including celebrated
commentaries (or bhashyas) on 10 Upanishads, the Brahma Sutra and the
Bhagavad Gita and wrote books like Upadesh Shastri, Vivek Chudamani
and Bhaja Govindum Stotra.
Ramanujacharya lived in the eleventh and twelfth century CE. He was one of the
most important exponents of the Sri Vaishnavism tradition within Hinduism. His
philosophical foundations for devotionals were influential to the Bhakti
movement. His Vishishtadvaita (qualified non-dualism) philosophy has
competed with the Dvaita (theistic dualism) philosophy of Madhvacharya, and the
Advaita (non-dualism) philosophy of Adi Shankaracharya. He wrote books such as
Sribhashya, Vedanta Dip, Gita Bhasya and Vedantasara. He and Adi
Shankaracharya were not contemporaries. So, statements 3 and 4 are not
correct.

So, only two of the above statements are correct.

Therefore, option (b) is the correct answer.

Knowledge Box
Shankaracharya or ‘teacher of the way of Shankara’, is a religious title used by
the heads of the four cardinal mathas or peeths believed to have been established
by Adi Shankara. According to tradition, they are religious teachers who belong to
a line of teachers going back to Adi Shankara himself.

Relevance: Recently, the Shankaracharyas of four mathas in India refused to attend the
inauguration of Ram Mandir.

Question 84. + 2.0 - 0.66

Doli-Puran, a historical ballad composed by Narrotam Das in Assamese, narrates events


related to which one of the following?

A. Paharias' Rebellion

B. Chuar Uprising

C. Sanyasi Revolt

  D. Patharughat Uprising

Question Analytics
2340 USERS 1217 USERS 1123 USERS 40.7 SECS

ATTEMPTED  SOLVED CORRECTLY  SOLVED INCORRECTLY  YOUR TIME

26.56 SECS 0.0 SECS

 AVG. SOLVING TIME  FASTEST SOLVING TIME

Explanation :

Doli-Puran is a historical ballad in Assamese. Composed by Narrottam Das, it


narrates events related to the Patharughat Peasant Resistance of 1894 that took
place in Assam. On 28 January 1894, more than a hundred peasants fell to the
bullets of the British army in Darrang. The unarmed peasants were peacefully
protesting against the increase in land revenue levied by the British when the
military opened fire.
The book, Doli Puran, was recovered in Udalguri district and was first published
in 1949. Narottam was an eyewitness to the uprising and described the events in
his song Doli (earth/soil). The unarmed peasants are said to have thrown clods of
soil at the British police who were firing at them.

Therefore, option (d) is the correct answer.


Question 85. + 2.0 - 0.66

In colonial India, the Delhi Durbars were organised by the Viceroys to:

  A. mark the coronation of the Monarch of Great Britain

B. celebrate the renewal of trade licence of the East India Company

C. honour friendly Princely States in British India

D. mark the transfer of Viceroyship to the newly appointed officer

Question Analytics
4007 USERS 3151 USERS 856 USERS 35.1 SECS

ATTEMPTED  SOLVED CORRECTLY  SOLVED INCORRECTLY  YOUR TIME

28.36 SECS 0.0 SECS

 AVG. SOLVING TIME  FASTEST SOLVING TIME

Explanation :

The Delhi Durbars were grand events organised by the Viceroys to mark the
coronations of Emperors or Empresses of Great Britain. Hence, these were also
known as the Coronation Durbars. Durbar, which means a 'court of a ruler' in
Persian, was adopted by the British from the Mughals. The idea was to come
across to the people of India as the 'heirs of the Mughals'.

Therefore, option (a) is the correct answer.

Knowledge Box

Delhi Durbar 1877: Queen Victoria in addition to her title of 'Queen of Great
Britain and Ireland' assumed the title of 'Empress of India'.
Delhi Durbar 1903: It marked the succession of Edward VII.
Delhi Durbar 1911: It marked the succession of King George V. It was historic for
two reasons; one it was the only one attended by the Emperor himself and
second, this was where the shifting of the Imperial Capital from Calcutta to
Delhi was announced.

Question 86. + 2.0 - 0.66

Consider the following:


1. Publication of Al-Hilal and Comrade newspapers

2. Visit of the Prince of Wales


3. Protest by Khudai Khidmatgars
4. Establishment of parallel governments

5. March on the Dharasana Salt Works

How many of the above events took place during the Civil Disobedience Movement
(CDM)?

A. Only two

 B. Only three

C. Only four

D. All five

Question Analytics
2696 USERS 1313 USERS 1383 USERS 11.0 SECS

ATTEMPTED  SOLVED CORRECTLY  SOLVED INCORRECTLY  YOUR TIME

37.76 SECS 0.0 SECS

 AVG. SOLVING TIME  FASTEST SOLVING TIME

Explanation :

Inspired by Mahatma Gandhi’s march to Dandi, the Civil Disobedience Movement


(CDM) of 1930 in India stretched to every corner of the country, touching even the
smallest districts.

Both Al Hilal (1912-1914) and Comrade (1911-1914) were prominent Urdu weekly
newspapers that played a significant role in shaping Indian public opinion during
the early 20th century. However, these were not associated with the CDM. So,
point 1 is not correct.
The visit of the Prince of Wales began on 17 November 1921 during the Non-
Cooperation Movement and not CDM. The day the Prince landed in Bombay was
observed as a day of hartal all over the country. He was greeted with empty streets
and downed shutters wherever he went. So, point 2 is not correct.
Khudai Khidmatgars participated actively in the CDM in 1930-31. They played a
significant role in the movement, particularly in the North-West Frontier Province
(NWFP). So, point 3 is correct.
Textile workers of Solapur in Maharashtra went on a strike during CDM and
along with other residents burnt liquor shops and other symbols of Government
authority such as railway stations, police stations, municipal buildings, law courts,
etc. The activists established a virtual parallel government which could only be
dislodged with martial law. So, point 4 is correct.
During the CDM, On May 21, 1930, Sarojini Naidu, Imam Sahib and Manilal
(Gandhi’s son) took up the unfinished task of leading a raid on the Dharasana
Salt Works. The unarmed and peaceful crowd was met with a brutal lathi-charge.
This new form of salt satyagraha was eagerly adopted by people in Wadala
(Bombay), Karnataka (Sanikatta Salt Works), Andhra, Midnapore, Balasore, Puri
and Cuttack. So, point 5 is correct.

So, only three of the above events took place during the Civil Disobedience
Movement (CDM).

Therefore, option (b) is the correct answer.

Question 87. + 2.0 - 0.66

Consider the following trees:

1. Mahogany
2. Ebony

3. Khejri

How many of the above are found in the Western Ghats?

A. Only one

  B. Only two

C. All three

D. None

Question Analytics
3610 USERS 2173 USERS 1437 USERS 9.2 SECS

ATTEMPTED  SOLVED CORRECTLY  SOLVED INCORRECTLY  YOUR TIME

18.1 SECS 0.0 SECS

 AVG. SOLVING TIME  FASTEST SOLVING TIME

Explanation :

Tropical Evergreen Forest: These forests are found in the western slope of the
Western Ghats, hills of the northeastern region and the Andaman and
Nicobar Islands. They are found in warm and humid areas with an annual
precipitation of over 200 cm and a mean annual temperature above 220 celsius.
Species found in these forests include rosewood, mahogany, aini, ebony, etc. So,
points 1 and 2 are correct.
Tropical Thorn Forests: These occur in the areas which receive rainfall less than
50 cm. These consist of a variety of grasses and shrubs. It includes semi-arid areas
of south west Punjab, Haryana, Rajasthan, Gujarat, Madhya Pradesh and Uttar
Pradesh. Important species found are babool, ber, and wild date palm, khair,
neem, khejri, palas, etc. So, point 3 is not correct.
Tropical Deciduous Forests: These are the most widespread forests in India. They
are also called the monsoon forests. They spread over regions which receive
rainfall between 70-200 cm. Tendu, palas, amaltas, bel, khair, axlewood, etc.
are the common trees of these forests.
Montane Forests: In mountainous areas, the decrease in temperature with
increasing altitude leads to a corresponding change in natural vegetation. In the
Himalayan region, between 1,500-1,750 m, Chir Pine is a very useful commercial
tree. Deodar, a highly valued endemic species, grows mainly in the western part
of the Himalayan range.

So, only two of the above are found in the Himalayas as well as the Western Ghats.

Therefore, option (b) is the correct answer.

Question 88. + 2.0 - 0.66

Consider the following statements:

Statement-I: Cirrus clouds help in the cooling of the surface of the Earth.

Statement-II: High-level thin clouds are transparent to shortwave radiation but absorb
the outgoing longwave radiation.

Which one of the following is correct in respect of the above statements?

A. Both Statement–I and Statement–II are correct and Statement–II is the


correct explanation for Statement–I

B. Both Statement–I and Statement–II are correct and Statement–II is not the
correct explanation for Statement–I

C. Statement–I is correct but Statement–II is incorrect

 D. Statement–I is incorrect but Statement–II is correct

Question Analytics
3528 USERS 1385 USERS 2143 USERS 12.7 SECS

ATTEMPTED  SOLVED CORRECTLY  SOLVED INCORRECTLY  YOUR TIME

41.42 SECS 0.0 SECS

 AVG. SOLVING TIME  FASTEST SOLVING TIME


Explanation :

High-level clouds occur above about 20,000 feet and are referred to as cirrus
clouds. Due to cold tropospheric temperatures at these levels, the clouds
primarily are composed of ice crystals and often appear thin, streaky, and white.
They are highly transparent to shortwave radiation but they readily absorb the
outgoing longwave radiation. They then emit the longwave radiation back to the
Earth's surface. So, statement 2 is correct.
The portion of the radiation thus trapped and sent back to the Earth's surface adds
to the shortwave energy from the Sun and the longwave energy from the air
already reaching the surface. The additional energy causes a warming of the
surface and atmosphere. The overall effect of the high thin cirrus clouds then is to
enhance atmospheric greenhouse warming. On the other hand, low, thick
clouds primarily reflect solar radiation and cool the surface of the Earth. So,
statement 1 is not correct.

So, Statement–I is incorrect but Statement–II is correct.

Therefore, option (d) is the correct answer.

Question 89. + 2.0 - 0.66

Which one of the following statements best describes the term 'population momentum'?

It is the phenomenon where a country witnesses an increase in young


 A.
population even if there is a decline in fertility rates.

It is a situation where the older population exceeds the population in the


B.
reproductive age.

It is a phenomenon where the population remains stagnant due to a balance


C.
between birth and death rates.

It is an increasing rate of population growth as a result of declining


 D.
mortality rates.

Question Analytics
3699 USERS 2211 USERS 1488 USERS 25.6 SECS

ATTEMPTED  SOLVED CORRECTLY  SOLVED INCORRECTLY  YOUR TIME

40.41 SECS 0.0 SECS

 AVG. SOLVING TIME  FASTEST SOLVING TIME

Explanation :
Population momentum is the tendency of a highly fertile population that has
been increasing rapidly in size to continue to do so for decades after the onset of
even a substantial decline in fertility. This results from the youthful age structure
of such a population.
These populations contain large numbers of children who have still to grow into
adulthood and the years of reproduction. Thus, even a dramatic decline in
fertility, which affects only the numbers at age zero, cannot prevent the
continuing growth of the number of adults of childbearing age for at least two or
three decades.
Eventually, as these large groups pass through the childbearing years to middle
and older age, the smaller numbers of children resulting from the fertility decline
lead to a moderation in the population growth rate.

Therefore, option (a) is the correct answer.

Relevance: The world’s population is expected to grow largely driven by a


phenomenon called population momentum.

Question 90. + 2.0 - 0.66

Which of the following evidence supports the theory that the Himalayas are still rising?

1. Frequent shallow earthquakes


2. Discovery of marine fossils

3. Youthful Himalayan rivers

4. Residual nature of mountains


Select the correct answer using the code given below:

A. 1 and 2 only

B. 3 and 4 only

 C. 1, 2 and 3 only

 D. 1, 2, 3 and 4

Question Analytics
4052 USERS 1654 USERS 2398 USERS 34.4 SECS

ATTEMPTED  SOLVED CORRECTLY  SOLVED INCORRECTLY  YOUR TIME

47.71 SECS 0.0 SECS

 AVG. SOLVING TIME  FASTEST SOLVING TIME

Explanation :
Due to the fierce pressure, the Indian plate is slowly being pushed below the
Eurasian Plate. This built-up pressure is released by sudden movements which
take place deep inside the Earth's crust. It releases a huge amount of energy
sending shock waves to the surface resulting in earthquakes. The swarms of
earthquakes and periods of quietude occur just before a giant earthquake
strikes the region showing that the Himalayas are still rising. So, point 1 is
correct.
The discovery of a hippopotamus skull at a high altitude in Ladakh indicates
that till recently the Himalayan region was a warm, low-lying swampy region.
The findings revealed that their presence at such heights highlights the tectonic
forces that lifted these sediments from a near-coastal marine environment to
these heights. Fossils of marine animals discovered at high altitudes in the
Himalayas indicate that these locations were once aquatic and submerged
throughout the early stages of mountain range construction. Their current height
above sea level suggests a significant uplift since fossilisation, indicating
continued vertical migration. So, point 2 is correct.
Evidence of Stone Age man in the higher reaches shows that the mountains'
rapid rise was more recent. It could have been witnessed by primitive man.
The Himalayan rivers are still in their (geomorphologically) youthful stage,
despite their great age. This ever-youthfulness of the Himalayan rivers is an
inherent character due to the continuing uplift of the terrains through which the
rivers flow. As the Himalayan terrain rose progressively, the rivers kept entering
their courses deeper and deeper. Over the long period of millions of years, deep
gorges or canyons with nearly vertical walls developed in the stretches of
impediments. Where the mountain barrier rose much faster and the rivers failed
to keep pace, they dropped in waterfalls and cascades. So, point 3 is correct.
Himalayas are fold mountains. Fold mountains are created where two or more
of Earth's tectonic plates are pushed together. At these colliding, compressing
boundaries, rocks and debris are warped and folded into rocky outcrops, hills,
mountains, and entire mountain ranges. Fold mountains are created through a
process called orogeny. Aravalli is a type of residual mountain. So, point 4 is not
correct.

Therefore, option (c) is the correct answer.

Knowledge Box

Hippopotamus flourished in India until the end of the Pleistocene era. They
may have disappeared owing to climate change, rapidly depleting water
sources and the disappearance of vegetation. It is estimated that the last glacial
period ending 14,000 years ago brought these changes in the climate.
There is no mention of hippopotamus anywhere in ancient Indian sculpture,
cave art, architecture, folk art or folklore.

Relevance: According to researchers, the Himalaya is a fragile mountain range that is


still rising as the Indian plate continues to push into the Asian plate.

Question 91. + 2.0 - 0.66

Which one of the following, famously known as the Pink Lake, is on the verge of
extinction due to high levels of pollution?

A. Lake Titicaca

B. Lake Chad

 C. Lake Retba

D. Lake Volta

Question Analytics
2946 USERS 1327 USERS 1619 USERS 11.7 SECS

ATTEMPTED  SOLVED CORRECTLY  SOLVED INCORRECTLY  YOUR TIME

22.96 SECS 0.0 SECS

 AVG. SOLVING TIME  FASTEST SOLVING TIME

Explanation :

Lake Retba, more commonly referred to as the Pink Lake, is located around
35km from the city of Dakar, Senegal. The lake is isolated from the sea by sand
dunes. Its fresh water comes from the seasonal water table in the dunes, which
are higher than the lake. The lake’s waters are virtually devoid of life, except
for a few microscopic algae and bacteria.
The Lake had extremely high levels of nitrates —chemical compounds —in May
2023: 12,491.71 to 15,394.75 mg/l compared with 5.82 to 12.05 mg/l in December
2022. The nitrate limit set by the World Health Organization (WHO) is 50 mg/l for
water to be considered potable. This sudden increase in nitrate levels is believed
to be associated with the establishment of hotels near the lake and ponds, and the
rapid, uncontrolled urbanisation of this northern part of the lake, which has no
sewage system.
The pink colouration of the lake is due to the proliferation of halophilic green
algae (living in a salty environment), Dunaliella salina, which contains red
pigments. The alga is associated with halophilic bacteria of the genus
Halobacterium. This microscopic alga’s resistance to salt comes from its high
concentration of carotenoid pigments, which protect it from light, and its high
glycerol content.

Therefore, option (c) is the correct answer.

Knowledge Box

Lake Titicaca: It is the largest freshwater lake in South America. It is located


high in the Andes mountains between Peru and Bolivia, divided by the Tiquina
Strait.
Lake Chad: It is a vast area of fresh water located in the middle of dunes which
covers territories in 4 countries: Nigeria, Cameroon, Niger and Chad. This lake
has been constantly evolving with respect to environmental conditions.
Lake Volta: It is an artificial lake in Ghana. The lake is formed by the Akosombo
Dam, which, begun in 1961 and completed in 1965, dammed the Volta River just
south of Ajena and created a lake extending upstream from the Akosombo Dam to
Yapei, beyond the former confluence of the Black Volta and White Volta rivers.

Relevance: Lake Retba’s waters are virtually devoid of life and are on the verge of
disappearing due to pollution and mining.

Question 92. + 2.0 - 0.66

Arrange the following cities located around the South China Sea in North to South
direction:

1. Taipei

2. Ho Chi Minh City

3. Manila

4. Singapore

Select the correct answer using the code given below:

A. 1-2-3-4

  B. 1-3-2-4

C. 2-1-3-4

D. 4-3-1-2
Question Analytics
3332 USERS 1130 USERS 2202 USERS 44.6 SECS

ATTEMPTED  SOLVED CORRECTLY  SOLVED INCORRECTLY  YOUR TIME

43.96 SECS 0.0 SECS

 AVG. SOLVING TIME  FASTEST SOLVING TIME

Explanation :

Taipei is a special municipality and seat of the government of Taiwan (Republic of


China).
Manila is the capital and chief city of the Philippines. The city is the centre of the
country’s economic, political, social, and cultural activity. It is located on the island
of Luzon and spreads along the eastern shore of Manila Bay at the mouth of the
Pasig River.
Ho Chi Minh City is the largest city in Vietnam. The city lies along the Saigon
River to the north of the Mekong River delta, about 50 miles (80 km) from the
South China Sea.
Singapore is a city-state located at the southern tip of the Malay Peninsula. It
consists of the diamond-shaped Singapore Island and some 60 small islets. The
main island is separated from Peninsular Malaysia to the north by Johor Strait.

So, the correct chronological order of the above cities from North to South
direction is 1-3-2-4.

Therefore, option (b) is the correct answer.


Relevance: Recently, the Philippines and Vietnam agreed to expand cooperation in the
South China Sea.

Question 93. + 2.0 - 0.66

A lower amount of carbon dioxide (CO2) gas in the atmosphere of a planet compared to

that of its neighbours can indicate:

A. absence of habitability

 B. presence of liquid water

C. terrestrial nature of the planet

D. increased volcanic activity

Question Analytics
3921 USERS 1068 USERS 2853 USERS 30.5 SECS

ATTEMPTED  SOLVED CORRECTLY  SOLVED INCORRECTLY  YOUR TIME

52.91 SECS 0.0 SECS

 AVG. SOLVING TIME  FASTEST SOLVING TIME

Explanation :

Habitability is a theoretical astronomical concept that means a celestial body is


capable of hosting and retaining liquid water on its surface. The habitable zone is
sometimes referred to as the Goldilocks zone.
Recently, an international team of researchers has found that a lower amount of
Carbon Dioxide (CO₂) gas in the atmosphere of a planet compared to that of its
neighbours could hint at the presence of liquid water on that planet. It is being
claimed that it can be used as a new 'habitability signature' and that it is a
“practical method for detecting habitability”. CO₂ is a strong absorber in the
infrared. By comparing the amount of CO₂ in different planets’ atmospheres, it can
be used to identify those planets with oceans, which make them more likely to be
able to support life.
Life on Earth accounts for 20 percent of the total amount of captured CO₂, with the
rest mainly being absorbed by the oceans. On another planet, this number
could be much larger. Oxygen can transform into ozone and it turns out ozone
has a detectable signature right next to CO₂.
Volcanic eruptions lead to large amounts of CO₂ emissions. Venus is a
terrestrial planet and is composed of more than 96 per cent CO₂.

Therefore, option (b) is the correct answer.

Relevance: Recently, an international team of researchers has developed a new method


to detect the potential habitability of exoplanets using carbon dioxide levels.

Question 94. + 2.0 - 0.66

Consider the following:

1. Gallium arsenide
2. Silicon

3. Boron

4. Germanium
5. Neon

6. Diamond

How many of the above elements can be used as semiconductors?

A. Only three

B. Only four

 C. Only five

D. All six

Question Analytics
3578 USERS 614 USERS 2964 USERS 9.7 SECS

ATTEMPTED  SOLVED CORRECTLY  SOLVED INCORRECTLY  YOUR TIME

27.73 SECS 0.0 SECS

 AVG. SOLVING TIME  FASTEST SOLVING TIME

Explanation :

A semiconductor is a substance that has specific electrical properties that


enable it to serve as a foundation for computers and other electronic devices. It is
typically a solid chemical element or compound that conducts electricity under
certain conditions but not others. This makes it an ideal medium to control
electrical current and everyday electrical appliances. They have properties that sit
between the conductor and insulator.
Gallium arsenide, germanium and silicon are some of the most commonly used
semiconductors. Silicon is used in the fabrication of electronic circuits. So, points
1, 2 and 4 are correct.
Gallium arsenide is a semiconductor compound used in some diodes, field-
effect transistors (FETs), and integrated circuits (ICs).
Elemental semiconductors include antimony, arsenic, boron, carbon, selenium,
sulphur and tellurium. Among wide bandgap semiconductors, diamond is
considered to be the ultimate semiconductor for applications in high-power
electronics due to its exceptional properties. So, points 3 and 6 are correct.
Neon is important to manufacture semiconductor chips but is not present in
the chips. It doesn't directly touch the silicon during manufacturing. It helps make
the Deep Ultraviolet (DUV) light used in the photolithographic process that
patterns semiconductors. So, point 5 is not correct.

So, only five of the above elements are used as semiconductors.

Therefore, option (c) is the correct answer.


Relevance: Recently, a novel laser slicing technique was invented for diamond
semiconductors.

Question 95. + 2.0 - 0.66

In biotechnology, ‘DnaA protein’ is used for:

 A. initiation of the DNA replication

B. joining breaks in the DNA

 C. identification of DNA polymorphism

D. arranging the DNA into specific sequences

Question Analytics
2742 USERS 1048 USERS 1694 USERS 29.2 SECS

ATTEMPTED  SOLVED CORRECTLY  SOLVED INCORRECTLY  YOUR TIME

28.07 SECS 0.0 SECS

 AVG. SOLVING TIME  FASTEST SOLVING TIME

Explanation :
The dnaA gene which encodes for the DnaA protein is considered a cold-inducible
protein and possesses both DNA binding/replication initiator properties and acts
as a global regulator of transcription. The DnaA protein is centrally involved in
the initiation of chromosomal and mini-chromosomal DNA replication and
appears to be important in the timing control of cell-cycle initiation. It also
autoregulates the dnaA gene and influences cell membrane structural properties.

Therefore, option (a) is the correct answer.

Relevance: Recently, scientists have uncovered how DnaA, the master key to DNA
replication, opens the door to bacterial growth.

Question 96. + 2.0 - 0.66

The term ‘Ergosphere Zone’, often seen in the news, refers to:

A. a region where the impact of solar flares is the strongest

B. boundary that marks the end of habitable zone in space

 C. a region in black hole where an object can enter and exit

D. point in space where matter moves in a perfect spiral due to earth's gravity

Question Analytics
3005 USERS 1516 USERS 1489 USERS 20.5 SECS

ATTEMPTED  SOLVED CORRECTLY  SOLVED INCORRECTLY  YOUR TIME

30.55 SECS 0.0 SECS

 AVG. SOLVING TIME  FASTEST SOLVING TIME

Explanation :

The formation of a black hole occurs when a massive star exhausts its nuclear
fuel, leading to a supernova explosion. The remaining core implodes under its
gravitational weight, creating a black hole.

Rotating black holes, also known as Kerr black holes, possess a distinctive feature
known as the ergosphere, which lies outside their outer event horizon.
Singularity is the point of infinite density and infinitesimal volume, at which
space and time become infinitely distorted according to the theory of General
Relativity.
The event horizon delineates a sphere around the singularity. Once an object
crosses this sphere, escape becomes impossible unless it somehow surpasses the
speed of light, an impossibility in classical physics. Just beyond this boundary, a
rotating black hole exhibits an ergosphere—an outer sphere where an object
can enter and exit if it attains sufficient speed, but still less than the speed of
light.

Therefore, option (c) is the correct answer.

Relevance: Recently scientists have suggested sending an object into the ergosphere of
a black hole.

Question 97. + 2.0 - 0.66

Consider the following statements with reference to solid-state batteries:


1. They are highly stable as they are resistant to changes in temperature.

2. Unlike lithium-ion batteries, they can be charged faster.


3. They have a lower risk of fire as they do not contain flammable organic solvents.
4. They can potentially be used in space devices.

How many of the above statements are correct?

A. Only one

B. Only two

C. Only three

 D. All four

Question Analytics
2818 USERS 1294 USERS 1524 USERS 18.3 SECS

ATTEMPTED  SOLVED CORRECTLY  SOLVED INCORRECTLY  YOUR TIME

35.55 SECS 0.0 SECS

 AVG. SOLVING TIME  FASTEST SOLVING TIME

Explanation :

A solid-state battery is a battery technology that uses a solid electrolyte instead of


a liquid electrolyte. They are characterised by being highly stable because they
are resistant to changes in temperature and can robustly endure high
temperatures and high voltages. So, statement 1 is correct.
Lithium-ion batteries typically have slower charging rates due to limitations in
ion mobility within the liquid electrolyte. Solid-state batteries have the potential
for faster charging due to improved ion conductivity in the solid electrolytes,
allowing for quicker charge transfer. So, statement 2 is correct.
One of the expected applications for solid-state batteries is electric vehicles.
Currently, electric vehicles use lithium-ion batteries. However, if they use solid-
state batteries, the risk of ignition due to accidents is expected to decrease since
they do not contain flammable organic solvents. So, statement 3 is correct.
It is also anticipated that use of solid-state batteries will include electronic device
backup power supplies and Internet of Things (IoT) sensors. If used on PCs or
smartphones, they should enable powerful operation for a longer time.
Furthermore, they can be expected to be used in aeroplanes and ships. Moreover,
as they are resistant to temperature changes across the spectrum from high to low
temperature, it can be expected that their applications will expand to include
devices used in outer space. So, statement 4 is correct.

So, all four of the above statements are correct.


Therefore, option (d) is the correct answer.

Knowledge Box

Relevance: Recently, Toyota announced plans to roll out a solid-state battery-powered


electric vehicle.

Question 98. + 2.0 - 0.66

“Mars solar conjunction” is a phenomenon that occurs when:

 A. Sun is between Earth and Mars

B. Sun is positioned nearest to Mars

C. Earth is between Mars and Sun

D. Sun, Mars and its moons align in a straight line

Question Analytics
3309 USERS 936 USERS 2373 USERS 12.7 SECS

ATTEMPTED  SOLVED CORRECTLY  SOLVED INCORRECTLY  YOUR TIME

34.65 SECS 0.0 SECS

 AVG. SOLVING TIME  FASTEST SOLVING TIME

Explanation :
Mars solar conjunction is a phenomenon which occurs when the Sun is between
Earth and Mars. It happens once every two years. The Sun expels hot, ionized gas
from its corona, which extends far into space.
During solar conjunction, this gas can interfere with radio signals when engineers
try to communicate with spacecraft at Mars (Red Planet), corrupting commands
and resulting in unexpected behaviour from deep space explorers.
Mars opposition occurs when Earth "catches up" to Mars and is positioned
between the Sun and the Red Planet. During Mars opposition from Earth, one
can see Mars rise in the east as the sun sets in the west, with the Red Planet being
visible in the skies all night long. Such opposition occurs roughly every 26 months.

Therefore, option (a) is the correct answer.


Relevance: Recently, NASA announced that it will stop sending commands to its fleet of
robotic probes on or orbiting the red planet Mars due to Mars's solar conjunction.

Question 99. + 2.0 - 0.66

Consider the following statements:

Statement-I: Multiple Independently Targetable Re-entry Vehicle (MIRV) reduces the


risk of interception by an anti-ballistic missile system.
Statement-II: MIRV allows a missile to carry multiple nuclear warheads which can be
independently targeted to different locations.
Which one of the following is correct in respect of the above statements?

Both Statement–I and Statement–II are correct and Statement–II is the


 A.
correct explanation for Statement–I

Both Statement–I and Statement–II are correct and Statement–II is not the
 B.
correct explanation for Statement–I

C. Statement–I is correct but Statement–II is incorrect

D. Statement–I is incorrect but Statement–II is correct

Question Analytics
3400 USERS 2585 USERS 815 USERS 52.1 SECS

ATTEMPTED  SOLVED CORRECTLY  SOLVED INCORRECTLY  YOUR TIME

29.59 SECS 0.0 SECS

 AVG. SOLVING TIME  FASTEST SOLVING TIME

Explanation :
Multiple Independently Targetable Re-entry Vehicle (MIRV) technology
consists of several reentry vehicles, each equipped to carry 2–10 nuclear
warheads. These warheads can be designated for various targets, spaced
hundreds of kilometers apart. Alternatively, multiple warheads can target a
single location. So, statement 2 is correct.
Deployment of MIRV can enhance first-strike proficiency for strategic forces and
also provide greater target damage for a given thermonuclear weapon payload.
Moreover, as it can release multiple warheads, it serves the purpose of a cluster
munition, as it reduces the number of missiles and launch facilities required. With
the MIRV, the effectiveness of an anti-ballistic missile system that relies on
intercepting individual warheads is also reduced. So, statement 1 is correct.

So, both Statement–I and Statement–II are correct and Statement–II is the correct
explanation for Statement–I.

Therefore, option (a) is the correct answer.

Relevance: Recently, the first flight test of Made-in-India Agni-5 missile with Multiple
Independently Targetable Re-entry Vehicle (MIRV) technology was conducted.

Question 100. + 2.0 - 0.66

‘Phthalates, Octoxynols and Nonoxynols’, causes of concern in recent times, are most
likely to be found in which one of the following?

A. Antibiotics

B. Performance-enhancing drugs

 C. Perfumes

 D. Food preservatives

Question Analytics
3152 USERS 856 USERS 2296 USERS 15.2 SECS

ATTEMPTED  SOLVED CORRECTLY  SOLVED INCORRECTLY  YOUR TIME

41.99 SECS 0.0 SECS

 AVG. SOLVING TIME  FASTEST SOLVING TIME

Explanation :

Phthalates, Octoxynols and Nonoxynols are chemicals widely used in the


manufacture of counterfeit perfumes and fragrances.
Phthalates are typically employed as solvents and stabilisers in perfumes. They
are known hormone disruptors linked to birth defects in male infants.
Octoxynols and nonoxynols are also concerning as they can break down into
persistent hormone disruptors.
The usage of such inferior and possibly adulterated cosmetics can lead to a
spectrum of adverse health reactions, ranging from mild skin irritations to
severe conditions such as anaphylactic shock and even organ failure. Due to their
low biodegradability, they persist in the environment, especially in aquatic
ecosystems and can encourage bioaccumulation.

Therefore, option (c) is the correct answer.

Relevance: BAN Toxics, a chemical and waste management advocacy group, has issued
a warning against the alarming rise in the sale of counterfeit perfumes.

You might also like